Head and Neck Tumors/Parotid, Anatomy

Pataasin ang iyong marka sa homework at exams ngayon gamit ang Quizwiz!

A 70-year-old man with a history of smoking and oromandibular cancer is scheduled to undergo extirpation, virtual planned fibular mandibular reconstruction, and postoperative radiation therapy. The patient strongly wants dental restoration. Which of the following best describes the earliest that dental implants can be safely placed? A) Immediately B) After radiation therapy C) After failure of dentures D) One year postoperaratively E) Never

A A 70-year-old man with a history of smoking and oromandibular cancer is scheduled to undergo extirpation, virtual planned fibular mandibular reconstruction, and postoperative radiation therapy. The patient strongly wants dental restoration. Which of the following best describes the earliest that dental implants can be safely placed? A) ImmediatelyB) After radiation therapyC) After failure of denturesD) One year postoperarativelyE) Never

Which of the following viruses is implicated in the pathogenesis of nasopharyngeal carcinoma? A) Epstein-Barr virus B) Hepatitis C virus C) Human herpesvirus 8 D) Human immunodeficiency virus E) Human papillomavirus

A All phases of the Epstein-Barr virus life cycle are associated with human disease. In immunocompromised individuals, infected cells increase in number, and eventually B-cell growth control pathways are activated, inducing transformation and leading to malignancies such as nasopharyngeal carcinoma, Burkitt lymphoma, post-transplant lymphomas, and gastric carcinomas. Human papillomavirus (HPV) is increasingly recognized as a pathogenic risk factor for oropharyngeal cancer development. Accumulating molecular and epidemiological data now show that high-risk types of HPV are responsible for a subset of oropharyngeal cancer. Oral verrucous and squamous cell carcinomas have been reported in patients infected with hepatitis C virus, and the infection has been found to be more prevalent in patients with oral lichen planus. Infection with HIV is not known to be directly pathogenic in malignant transformation, but rather it increases the susceptibility to opportunistic infections and viral-promoted cancers. Human herpesvirus 8, also known as Kaposi sarcoma-associated herpesvirus, has been found in nearly all tumors in patients with Kaposi sarcoma.

A 40-year-old woman comes to the office because of a 5-year history of firm, painless swelling of the upper jaw that has increased progressively in size. CT scan is performed, and the lesion is shown. Resection is performed. Pathologic examination shows odontogenic epithelial islands bordered by palisading columnar cells. No invasion into the surrounding tissues is noted. Which of the following is the most likely diagnosis? A) Ameloblastoma B) Fibrous dysplasia C) Nasopharyngeal angiofibroma D) Osteosarcoma E) Squamous cell carcinoma

A Ameloblastomas are benign, locally invasive, odontogenic tumors accounting for 1% of tumors of the jaw and 10% of odontogenic tumors. Approximately 80% occur in the mandible and 20% occur in the maxilla. The peak incidence is in the third and fourth decades but may also arise in children and adolescents. Ameloblastomas may be radiographically found to be unilocular or, more commonly, multilocular with a "soap bubble" or "honeycomb" appearance. Treatment may be with enucleation and curettage or more radical resection. In rare cases, metastatic ameloblastoma and ameloblastic carcinomas have been reported. Fibrous dysplasia is a benign hamartomatous lesion that has a diffuse, "ground-glass" appearance on x-ray studies. It is usually treated conservatively with shaving and re-contouring of the bone. Squamous cell carcinoma is usually associated with a painful mucosal lesion. Radiographically, bony invasion may be noted in locally advanced cases. Osteosarcomas are aggressive malignancies of mesenchymal origin that exhibit osteoblastic differentiation. They are the most common primary bony cancer. Their radiographic appearance is variable and may include nonspecific destruction of the bone similar to a carcinoma, mottled ossification, similar to fibrous dysplasia, but without well-defined borders, or lamellar ossification (sheets of neo-osteogenesis). Nasopharyngeal angiofibromas are benign but locally invasive vascular tumors that occur almost exclusively in male adolescents. Their symptoms include nasal obstruction but can eventually cause facial asymmetry and eye displacement, as they grow from the region of the sphenopalatine foramen first into the nasopharynx and choanae then into the paranasal sinuses, pterygopalatine and infratemporal fossae, orbit, and even the intracranial cavity. Radiographically, th

A 17-year-old boy is diagnosed with an infected molar and scheduled for dental extraction. Before the day of surgery, he is brought to the emergency department because of drooling, protruding tongue, bilateral woody edema of the submandibular region, and tenderness of the neck. Which of the following is the most likely diagnosis? A) Ludwig angina B) Periapical abscess C) Peritonsillar abscess D) Primary mononucleosis E) Reactive lymphadenopathy

A Drooling, protruding tongue, and woody edema (non-fluctuant) of the submandibular region are classic signs for Ludwig angina, or deep space infection of the floor of the mouth. The source is frequently dental periapical abscess, often molar in origin where the mandible bone is thinner, allowing the infection to spread to the floor of the mouth. Treatment includes ICU monitoring of the airway for possible impending intubation, antibiotics, and surgical drainage (and in this case, extraction of the tooth as well).Mononucleosis is not characterized by woody edema. Peritonsillar abscess is more frequently unilateral with lateral pharyngeal space symptoms without woody edema. Periapical abscess of the tooth involves localized symptoms without generalized bilateral submandibular symptoms, although this can eventually lead to Ludwig angina.Reactive lymphadenopathy is the result of head and neck infection and does not result in the symptoms described in this scenario.

A 60-year-old woman with a history of squamous cell carcinoma of the scalp treated with resection, skin grafting, and total scalp radiation therapy is evaluated for osteoradionecrosis of the cranial vertex. After full-thickness debridement, which of the following is the most appropriate method for soft-tissue reconstruction? A) Coverage with a free latissimus dorsi musculocutaneous flap B) Coverage with a pedicled trapezius musculocutaneous flap C) Coverage with a scalp rotation flap D) Negative pressure wound therapy E) Skin grafting

A In the patient described, with a history of previous radiation therapy and a full-thickness defect, a free latissimus dorsi musculocutaneous flap would be the best choice for soft-tissue reconstruction. Skin grafting over an implant cranioplasty would not be a suitable option in this setting. Skin grafts could be considered in nonradiated, partial-thickness defects of the scalp or for coverage of scalp rotation flap donor sites. A pedicled trapezius musculocutaneous flap would not reach the cranial vertex without undue tension and is better suited for full-thickness occipital defects. Scalp rotation flaps are ideal for defects up to 8 cm in diameter in a nonradiated scalp. The patient's history of previous radiation therapy would likely compromise the viability of a large rotation flap in this setting. Negative pressure wound therapy over an implant cranioplasty in a radiated field would not allow for soft-tissue healing and coverage.

Which of the following factors is most likely to increase the risk for osteoradionecrosis secondary to radiation therapy? A) Dental caries B) Edentulous mandible C) Oral thrush D) Osseointegrated implants E) Radiation dose of 3500 cGy

A Osteoradionecrosis (ORN) of the mandible is uncommon but can occur in up to 10% of patients after undergoing radiation therapy for oral cancers. The risk increases once radiation doses exceed 6500 cGy. Most reports of ORN have dental caries and extraction sites as precipitating factors. Periodontal disease can also lead to ORN. After undergoing radiation therapy, patients can develop oral candidiasis and xerostomia, and they may also have edentulous mandibles with dental implants after reconstruction. However, these do not increase the risk for ORN. Surgical resection and hyperbaric oxygen therapy are the mainstays of treatment.

A 45-year-old man with mucoepidermoid carcinoma of the left parotid gland undergoes parotidectomy. Which of the following complications is most likely in this patient? A) Gustatory sweating B) Hyperacusis C) Loss of taste D) Tardive dyskinesia E) Tearing while eating

A Parotidectomy is a surgical procedure performed for benign and malignant tumors of the parotid gland and, uncommonly, for hypersalivation. Specific risks for parotidectomy include facial nerve injury, sialocele formation, and gustatory sweating. Gustatory sweating (or Frey syndrome) results when there is aberrant innervation of cutaneous sweat glands in the cheek skin overlying the parotid bed resulting in sweating and flushing during eating. Loss of taste to the anterior two thirds of the tongue results from injury to the chorda tympani, which arises from the facial nerve as it crosses the tympanic cavity and joins with the lingual nerve, not from injury to the facial nerve within the parotid gland. The stapedius is a small muscle in the middle ear that restricts movement of the stapes bone in response to loud noises. Hyperacusis may occur when the facial nerve is interrupted intracranially, before it exits the stylomastoid foramen and becomes extracranial. Tearing while eating (crocodile tear syndrome or Bogorad syndrome) is an uncommon consequence of nerve regeneration subsequent to Bell's palsy in which efferent fibers from the superior salivary nucleus become improperly connected to nerve axons projecting to the lacrimal glands causing tears while smelling foods or eating. Tardive dyskinesia is a disorder resulting in involuntary repetitive movements including grimacing, tongue and lip movements, and excessive blinking that usually occur as side effects of antipsychotic drugs. It is unrelated to facial nerve dysfunction.

A 37-year-old man comes to the office for evaluation of whitish papules of the oral mucosa 3 weeks after undergoing successful face transplantation encompassing the maxilla, palate, oral mucosa, and overlying skin. Examination shows that the patient is afebrile and has no leukocytosis. No changes have been made to the initial acute postoperative immunomodulation therapy. Which of the following is the most appropriate initial management of this patient? A) Apply topical nystatin B) Biopsy for allograft rejection C) Increase the dose of tacrolimus D) Initiate oral amoxicillin therapy E) Reexamine the patient in 4 weeks

A Post-transplant infections can be divided into three phases: early postoperative infections (0 to 2 months), intermediate (2 to 9 months), and late (greater than 9 months). Because of the variation in tissue type and anatomy between solid organ transplantation and composite tissue allograft transplantation, there are differences between types of infection. In all transplant patients, cytomegalovirus (CMV) infection/reactivation is the most common infectious complication. As a result, the protocol for composite tissue allograft recipients includes CMV prophylaxis. In this particular scenario, facial transplantation is unique in that there are oral and sinus cavities with colonization with native flora. Candida is the most ubiquitous and most common flora. Treatment is topical nystatin and clotrimazole. Although there is constant wariness for rejection, this does not occur commonly in the immediate postoperative period. Biopsy would be warranted if there is no improvement with topical antifungals. As there is no indication of acute rejection, increasing tacrolimus is not an appropriate initial treatment. Amoxicillin is a bactericidal antibiotic and is not an appropriate treatment for this fungal infection.

A 45-year-old woman comes to the office because of a painful 4-cm left parotid mass. Physical examination shows weakness of the left facial muscles. CT scan of the chest shows multiple lung nodules consistent with metastases. Parotidectomy is performed, and pathologic examination of the gland shows a cribriform ("Swiss cheese") pattern of cells with perineural invasion. Which of the following is the most likely diagnosis? A) Adenoid cystic carcinoma B) Lymphangiosarcoma C) Mucoepidermoid carcinoma D) Pleomorphic adenoma E) Warthin tumor

A Salivary gland tumors are relatively rare and make up about 3 to 4% of all head and neck neoplasms. The majority of salivary gland tumors (approximately 80%) originate in the parotid gland. Approximately 80% of parotid gland tumors are benign. Malignant tumors are associated with facial paralysis and pain, although they may also be asymptomatic. Malignant tumors may also metastasize to the regional lymph nodes and to distant sites. Pleomorphic adenoma, also known as benign mixed tumor, is the most common benign tumor of the parotid gland. This tumor is histologically characterized by epithelial and connective tissue elements, with stellate and spindle cells interspersed with a mixoid background. Warthin tumor (papillary cystadenoma lymphomatosum) is the next most common tumor of the parotid gland and is also benign. Warthin tumors predominantly occur in males and are bilateral in 10% of patients. Histologically, they are characterized by papillary cysts and mucoid fluid as well as nodules of lymphoid tissue. Mucoepidermoid carcinoma is the most common malignancy of the parotid gland and the second most common malignancy of the submandibular and minor salivary glands. Mucoepidermoid carcinomas contain two major elements: mucus-producing cells, and epithelial cells of the epidermoid variety. Low-grade tumors are associated with a predominance of mucus-secreting cells lining cysts and intervening nests of well-differentiated epidermoid cells. High-grade tumors show few or no mucus-producing cells and the epidermoid cells are poorly differentiated. Intermediate-grade tumors are defined by less cyst formation than low-grade tumors with nests of epidermoid and less differentiated intermediate cells. The biologic behavior of mucoepidermoid carcinomas correlates with their histologic grade. Adenoid cystic carcinoma is the s

A 59-year-old woman who has worked in rubber manufacturing for 35 years is referred by her primary care physician because of a firm, nontender, 1-cm mass with ulceration on the hard palate between the central incisors and the incisive foramen. This suspected minor salivary gland tumor is most likely to drain into which of the following nodal basins? A) Submandibular nodes B) Parathyroid nodes C) Parotid nodes D) Posterior triangle nodes E) Occipital nodes

A The area of the lips, gums, teeth, tongue, and anterior hard palate will drain to the submental and submandibular region (Level I). Levels IA (submental) and IB (submandibular) are separated by the anterior digastric muscle. Group II drains the naso-/oro-/hypopharynx and parotid and is called the upper jugular group. Group III ( middle jugular group) drains the naso-/oro-/hypopharynx and larynx. The lower jugular group (level IV) drains the larynx, cervical esophagus, and hypopharynx. Group V is the posterior triangle group and drains the naso- and oropharynx. Group VI is the anterior central group below the hyoid and above the sternal notch. The group drains the thyroid, parathyroid, cervical esophagus, and larynx. More posteriorly the hard palate and soft palate will drain to the retropharyngeal space and deep cervical nodes.

A 58-year-old man is evaluated because of floor-of-mouth cancer that is invading the mandible. A segmental mandibulectomy and reconstruction with an osteocutaneous free flap that includes bone from the lateral border of the scapula are planned. The vascular pedicle supplying this flap is based on which of the following arteries? A) Circumflex scapular B) Dorsal scapular C) Lateral thoracic D) Thoracoacromial E) Transverse cervical

A The circumflex scapular artery, which is a branch of the subscapular artery, supplies blood to the lateral and medial borders of the scapular bone. The scapula free flap was first described in 1978 by Saijo. It can be harvested as part of a chimeric flap that includes other tissues supplied by the subscapular arterial system, such as the latissimus dorsi muscle, serratus anterior muscle, and scapular or parascapular skin. The tip of the scapula receives its blood supply from the angular branch of the thoracodorsal artery and has also been utilized as a pedicle for the inferior portion of the scapula. While the scapular bone is not as thick as the fibula bone, it provides adequate stability for mandibular reconstruction. Cutaneous branches of the circumflex scapular artery supply the scapular and parascapular skin and, therefore, a skin flap can be harvested simultaneously to close soft-tissue defects. The transverse cervical artery and the dorsal scapular artery primarily supply the trapezius muscle and overlying skin. The thoracoacromial artery supplies the pectoralis major muscle and overlying skin. The lateral thoracic artery supplies both the lateral portion of the pectoralis major muscle and the skin in the axillary region.

A 32-year-old man is evaluated because of painless swelling of the right cheek 2 months after being assaulted with a knife and sustaining a laceration across the mid cheek. The laceration was sutured by the emergency department physician assistant. Which of the following is the most appropriate management at this time? A) Drainage of the collection and administration of antisialagogues B) Open debridement with skin grafting C) Surgical exploration with reanastomosis of the injured duct D) Total parotidectomy with facial nerve preservation E) Ultrasound-guided needle aspiration only

A The correct response is Option A. Any penetrating injury on a line from the tragus to the mid upper lip may injure the parotid duct or gland. The parotid system can be divided into 3 regions: Region A: glandRegion B: duct superficial to the masseter muscleRegion C: duct from the masseter to where it enters the mouth opposite the 2nd maxillary molar Additionally, the buccal branch of the facial nerve often runs with the duct. It can cross the superficial layer of the masseter after leaving the parotid gland. Injury manifests as weakness of the upper lip on animation. Injury not repaired acutely can manifest later as salivary fistulae or sialoceles. Most can be managed conservatively with drainage, pressure, and antisialagogues. Trying to find and repair the duct late after the injury is difficult and may not lead to resolution of the symptoms. This is usually reserved for early injury and repair. If these fail, then a superficial parotidectomy can be considered for recalcitrant salivary collections.

A 60-year-old man is evaluated for a 6-cm ameloblastoma of the right maxilla. Reconstruction using an osteocutaneous iliac crest free flap is planned. Which of the following arteries supplies arterial blood to this flap? A) Deep circumflex iliac B) Deep inferior epigastric C) Descending genicular D) Lateral circumflex femoral E) Peroneal

A The deep circumflex iliac artery is the major blood supply to the iliac crest free flap. It gives rise to periosteal branches and nutrient endosteal branches that supply the iliac crest bone posterior to the anterior superior iliac spine. It also gives rise to an ascending branch that supplies the internal oblique muscle and several musculocutaneous perforators that supply the overlying skin, allowing a myo-osseous or osteocutaneous free flap to be harvested, respectively. Use of the iliac crest osteocutaneous free flap has been described by several authors for maxillary as well as mandibular reconstruction, and the bone itself provides ample stock for accommodating osseointegrated implants for dental restoration. The peroneal artery is the blood supply to the fibula free flap. Use of this flap is contraindicated when the peroneal artery contributes markedly to the blood supply of the distal lower extremity. The descending genicular artery is a branch of the superficial femoral artery and is the blood supply to the medial femoral condyle osseous free flap. Alternately, the medial superior genicular artery, another branch of the superficial femoral artery, can be used to supply this flap, but the pedicle is shorter. The descending branch of the lateral circumflex femoral artery is the blood supply to the anterolateral thigh free flap, which is a cutaneous perforator flap. The deep inferior epigastric artery is the blood supply to the rectus abdominis musculocutaneous free flap or the deep inferior epigastric perforator flap.

The periodontal ligament anchors the teeth to the surrounding alveolar bone through attachment to which of the following structures? A) Cementum B) Dentin C) Enamel D) Gingiva E) Neurovascular bundle

A The dental root is covered by a mineralized layer called cementum. This is anchored to the surrounding alveolar bone by the periodontal ligament (periodontal membrane), which includes Sharpey fibers, to firmly anchor the tooth to the bone as well as sensory fibers. This facilitates the periodontal ligament's functions of stabilizing the tooth, serving as a shock absorber, and allowing proprioception. The bulk of the dental root is composed of dentin, but it does not directly attach to the surrounding bone. The neurovascular bundles enter the tooth at the root apex and provide sensation and blood supply, but they are not a strong mechanical source of attachment. The enamel is the mineralized cover of the dental crown. It is not attached to the free gingiva, which attaches to the periodontal ligament at the cementoenamel junction. The gingiva does not directly attach to the bone.

A 47-year-old man is referred for examination of a composite defect of the mandible that extends from the right mandibular angle to the left mandibular angle. Which of the following flaps is most appropriate for reconstruction in this patient? A) Fibula B) Pectoralis C) Radial forearm D) Rectus E) Scapula

A The fibula flap is the most appropriate option in this case because a long section of bone requiring multiple osteotomies is needed. The fibula flap can provide 18 to 20 cm of bone and has both an endosteal and periosteal blood supply enabling shaping of the bone with multiple osteotomies. In addition, a skin paddle can be harvested with the flap to reconstruct the floor of mouth defect. The scapula and radial forearm flaps also provide bone and soft tissues; however, these flaps will not provide a long enough bone segment and cannot be reliably osteotomized in multiple locations. The rectus and pectoralis flaps are soft-tissue flaps, and their use in this case would result in marked deformity because the anterior arch has been resected.

Which of the following structures contributes to the formation of the mandibular body and ramus? A) First branchial arch B) First branchial cleft C) Second branchial arch D) Second branchial cleft

A The first branchial arch contributes to the formation of the mandible, the tragus, and the anterior helix. The first branchial cleft gives rise to the external auditory canal. The second branchial arch contributes to the formation of the majority of the external ear-the antitragus, remainder of the helix, antihelix, and crura all arise from the second branchial arch. The second branchial cleft is typically obliterated during development, but may persist in the form of a second branchial cleft cyst.

Which of the following structures contributes to the formation of the tragus? A) First branchial arch B) First branchial cleft C) Second branchial arch D) Second branchial cleft

A The first branchial arch contributes to the formation of the tragus and anterior helix. The first branchial cleft is incorrect. It gives rise to the external auditory canal. The second branchial arch is incorrect. It contributes to the formation of the majority of the external ear - the antitragus, remainder of the helix, antihelix, and crura all arise from the second branchial arch. The second branchial cleft is incorrect. It is typically obliterated during development, but may persist in the form of a second branchial cleft cyst.

A 64-year old woman comes to the office because of a nonhealing radiated scalp wound. Medical history includes resection of invasive basal cell carcinoma of the scalp, reconstruction with a scalp rotation flap, and high-dose postoperative radiation therapy (60 Gy) 10 years ago. Clinical examination shows a full-thickness wound consisting of erythematous, ulcerated, and necrotic skin, and exposed, foul-smelling skull at the base of the wound. Which of the following is the most appropriate next step in management of this patient? A) Biopsy of the wound B) Craniectomy with free flap reconstruction C) MRI D) Resection of involved scalp with split-thickness skin grafting E) Vacuum-assisted closure (VAC)

A The first step in managing this patient is biopsy of the wound to rule out cancer recurrence. Although the diagnosis is most likely osteoradionecrosis of the skull, one would not proceed with the next steps of management until recurrence of cancer is ruled out. In this patient, the management sequence would include a biopsy to rule out cancer recurrence, followed by CT scan to delineate the extent of the skull involvement. MRI would not delineate the extent of the bony involvement. The rates of osteoradionecrosis occurrence vary in the literature (from 1.8 to 37%). Although the rate and severity of osteoradionecrosis are most consistently associated with doses of radiation exceeding 50 Gy, there are reports of osteoradionecrosis in patients who received doses as low as 30 Gy. This patient would require extensive craniectomy by a neurosurgeon to debride the wound of necrotic bone and, in most cases, reconstruction with free tissue transfer. Vacuum-assisted closure would not be a viable option for this patient, nor would resection of the scalp with split-thickness skin grafting.

The inferior oblique muscle of the orbit is innervated by which of the following cranial nerves? A) Oculomotor (III) B) Trochlear (IV) C) Trigeminal (V) D) Abducens (VI) E) Facial (VII)

A The inferior oblique muscle receives its nerve supply from the oculomotor nerve, or cranial nerve III. The other voluntary muscles within the orbit that receive their innervation from the oculomotor nerve are the levator palpebrae superioris, superior rectus, medial rectus, and inferior medial rectus muscles. The superior oblique muscle is innervated by the trochlear nerve (cranial nerve IV). The lateral rectus muscle is innervated by the abducens nerve (cranial nerve VI).The inferior oblique is the only one of these muscles that does not arise from the apex of the orbit. It originates from the medial floor of the orbit just posterior to the infraorbital rim. It runs laterally, posteriorly, and upward, crossing inferior to the inferior rectus, and inserting on the posterior half of the globe beneath the lateral rectus insertion. Its action is to elevate the globe, producing upward gaze of the pupil. Risk of iatrogenic injury to the inferior oblique is greatest with transconjunctival surgical approach to the orbit. Therefore, with this procedure, care must be exercised to place the periosteal incision along the anterior aspect of the infraorbital rim.

A 23-year-old man comes to the office because of a 1-year history of painless swelling, asymmetry, and loss of interdental relationships on the right side of the jaw. Physical examination shows crowding of the right-sided first and second molars and premolar dentition. The third molar has not erupted; in its place there is a palpable firm enlargement of the mandible. Panoramic x-ray study (Panorex) shows a 3-cm radiolucent unilocular cyst. Percutaneous biopsy of the cyst shows nonkeratinizing stratified squamous epithelium. Which of the following is the most likely diagnosis in this patient? A) Dentigerous cyst B) Gingival cyst C) Gorlin cyst D) Primordial cyst E) Radicular cyst

A The most appropriate answer is dentigerous cyst. This type of cyst develops in the context of an unerupted tooth, which can be seen below the cyst. The cyst is lined with benign nonkeratinizing epithelium and is caused by degeneration of the enamel reticulum of the unerupted tooth. It is the second most common type of jaw cyst. Two thirds occur in the mandible. Gingival cysts appear most commonly on alveolar ridges of infants but can also rarely appear in adults. Their origin relates to rests of dental lamina, and, unlike dentigerous cysts, these contain keratin. Clinically, they are soft and fluctuant, and range in size between 1 and 15 mm. Primordial cysts and odontogenic keratocysts are equivalent. They develop from rests of dental lamina and basal cell hamartomas. Therefore, unlike dentigerous cysts, these are lined with a keratinizing stratified squamous epithelium that is sometimes dysplastic. Their size range is 1 to 9 cm, and pain is a common symptom. Gorlin syndrome includes the association of multiple odontogenic keratocysts with multiple basal cell carcinomas, nasal deformity, skeletal abnormalities, calcification of the falx cerebri, and palmar or plantar pits. Calcifying odontogenic (Gorlin) cysts are distinct from those above because they may be part cystic and part neoplastic. The histology shows features similar to the calcifying epithelioma of Malherbe (epithelium undergoing keratinization and calcification), and ameloblastic proliferations. Radiographically, they contain various amounts of radiopaque (calcified) material and are usually located anterior to the first molars ranging in size from 1 to 8 cm. Radicular cysts are the most common type of jaw cysts and develop at the apex of a nonviable erupted tooth, from epithelial rests of Malassez in the periodontal ligament. As these cysts are inflamm

A 37-year-old man sustains a deep laceration of the cheek from a broken bottle resulting in injuries to the facial (VII) nerve and the parotid duct. The nerve is repaired, and the parotid duct is repaired just distal to the masseter muscle over a stent. Treatment with ampicillin-sulbactam is started. Two days later, there is marked swelling and fluctuance in the cheek but no erythema or fever. Which of the following is the most appropriate next step in management? A) Aspiration and application of compression dressings B) External incision and placement of a passive drain C) Intraoral incision and placement of a passive drain D) Reexploration and revision of the parotid duct repair E) Reoperation for ligation of the parotid duct proximal to the site of injury

A The patient described has a sialocele, either from a leak at the site of the parotid duct repair or from direct injury to the gland. An abscess is less likely, given the prophylactic antibiotic treatment, no erythema, and no fever. Most occurrences of sialocele after parotid duct injury or repair will resolve with compression with or without repeated aspiration. Anticholinergic medications may help by decreasing salivary flow. External drainage of a sialocele may result in a fistula and should be avoided. Spontaneous external fistulas after parotid duct repair may be treated with intraoral drainage and compression. If the sialocele persists, excision of the cyst or pseudocyst may be required, with intraoral drainage over a stent, but this would not be appropriate initial management. Attempts to revise the parotid duct repair would not likely be fruitful because of postoperative inflammation and unnecessary if an adequate stent were in place and compression were applied. Ligation is indicated for initial management of proximal injuries to the parotid duct. Significant swelling of the parotid gland will generally occur after this procedure but typically resolves rapidly with atrophy of the gland.

A 44-year-old woman has a malignancy of the nasopharynx. She does not smoke cigarettes or drink alcohol. Which of the following viruses is the most common cause of this patient's malignancy? A) Epstein-Barr virus (EBV) B) Herpes simplex virus (HSV) C) Human immunodeficiency virus (HIV) D) Human papillomavirus (HPV) E) Human T-lymphotrophic virus-1 (HTLV-1)

A There is ongoing investigation into the pathogenesis of infectious agents in cancer formation. Although increasing attention is focused on the relationship between HPV and head and neck cancers, HPV strains 16 and 18 are associated with the oropharynx, not nasopharynx location. In contrast, the Epstein-Barr virus (EBV) is associated specifically with nasopharyngeal cancers as well as Burkitt lymphoma. The other viruses listed are not routinely associated with head and neck cancers.

A 40-year-old woman undergoes surgery for a right parotid tumor observed to be invading the facial nerve. During surgery, a segment of facial nerve is resected, resulting in a 5-cm gap from the main trunk to the distal branches. Which of the following is the most appropriate treatment for this patient? A) Cable nerve graft repair B) Cross-facial nerve grafting C) Masseteric nerve to facial nerve transfer D) Reconstruction with an innervated gracilis muscle free flap E) Temporalis tendon transfer

A When a facial nerve has been divided or resected, the best outcomes for regaining function are usually obtained from direct repair or cable nerve grafting. Facial motor nerve recovery is observed even in cancer patients who have preoperative weakness and who receive postoperative radiation therapy. When this is not possible, for example, when the nerve has been resected very proximally, up to the intracranial portion of the nerve, cross-facial nerve grafting between redundant branches of the normal contralateral nerve and the distal facial nerve stumps of the paralyzed side can produce excellent results with spontaneous symmetrical facial movement. In a patient with long-standing facial paralysis, cable nerve grafting or cross-facial nerve grafting will not be successful because the motor endplates of the distal facial nerve have degenerated. Performing a nerve transfer from a donor nerve, such as the masseteric (V), spinal accessory (XI), or hypoglossal (XII) nerves can provide facial tone and symmetry at rest, and, in some cases, volitional movement with training. When nerve repair or nerve transfers from the contralateral face or donor nerves are not feasible, innervated free muscle flap transfers can restore facial movement to the lower face. Muscles commonly used for facial reanimation include the gracilis, pectoralis minor, serratus anterior, and latissimus dorsi, due to their thinness, good excursion, and low donor site morbidity. In addition to a microvascular anastomosis, an epineural nerve repair is performed either to a cross-facial nerve graft or a donor cranial nerve such as the masseteric nerve. In patients who are not candidates for free muscle transfer or who refuse it, temporalis tendon slings can be used to suspend the face, usually at the oral commissure, to improve facial symmetry and function.

A 35-year-old woman undergoes surgical resection of a left parotid gland malignancy. The facial nerve was resected with the tumor, leaving a 5- to 7-cm gap between the proximal nerve stump and the distal nerve branches. Which of the following is the most appropriate treatment? A) Cable nerve grafting B) Cross-facial nerve grafting C) Hypoglossal nerve to facial nerve transfer D) Innervated gracilis muscle free flap reconstruction E) Nerve repair with a conduit

A When a facial nerve has been divided or resected, the best outcomes for regaining function are usually obtained from direct repair, or cable nerve grafting when too great a distance for direct repair separates the nerve ends. While autologous nerve grafts from "expendable" donor nerves, such as the great auricular nerve or sural nerve, have long been the gold standard, nerve repair using biologic or synthetic nerve conduits has also produced reasonable results, in some series equivalent to cable nerve grafts. Conduit nerve repair has the advantage of having no donor site morbidity. However, the length of the gap between the proximal and distal cut nerve ends is usually limited to less than 3 cm for the best chances of nerve recovery. When direct repair or cable nerve grafting is not possible—for example, when the nerve has been resected very proximally up to the intracranial portion of the nerve—cross-facial nerve grafting between redundant branches of the normal contralateral nerve and the distal facial nerve stumps of the paralyzed side can produce reasonable results with spontaneous symmetrical facial movement. Performing a nerve transfer from a donor nerve, such as the masseteric (V), spinal accessory (XI), or hypoglossal (XII) nerve can provide facial tone and symmetry at rest, and, in some cases, volitional movement with training. A temporary nerve transfer to these nerves is sometimes performed as a "babysitter" procedure while awaiting axonal growth through cross-facial nerve grafts. When nerve repair or nerve transfers from the contralateral face or donor nerves are not feasible, such as after motor endplate degeneration has occurred in the facial muscles, innervated free muscle flap transfers can restore facial movement to the lower face. Muscles commonly used for facial reanimation include the

A 65-year-old woman is scheduled to undergo reconstruction of a total laryngopharyngectomy defect. Use of which of the following is most likely to promote intelligible postoperative tracheoesophageal speech? A) Anterolateral thigh flap B) Gastric pull-up C) Jejunal flap D) Pedicled muscle flap E) Pedicled muscle flap

A When dealing with a near or total circumferential defect of the laryngopharyngeal unit, function must be considered as well as incidence of strictures and fistulas. Many of these patients receive a tracheoesophageal puncture prosthesis (TEP) for speech and many are quite intelligible after rigorous therapy. Speech with fasciocutaneous flaps, such as the anterolateral thigh flap and the radial forearm, is consistently better than with jejunal free flaps. In one study, a direct comparison between these flaps was performed, and 78% of patients used tracheoesophageal speech for conversation when reconstructed with an anterolateral thigh free flap compared with only 25% following a jejunal free flap. The distensibility and mucous production in reconstructions using a jejunal free flap appears to be responsible for the low-pitched, "wet" speech. There is some evidence that intensive speech rehabilitative programs can produce excellent results with jejunal flaps. A pedicled muscle flap is typically used to cover laryngeal or laryngopharyngeal closures, especially in radiation salvage cases. A silastic stent is usually used as salivary diversion while the reconstruction is healing. During its occupancy, no TEP speech can be generated. A gastric pull-up is usually used when a large portion of the cervical esophagus is involved with tumor, and thus a laryngopharyngoesophagectomy is performed so that the anastomosis of the swallowing system is cephalad to the thorax.

A 59-year-old man is referred to the office with a diagnosis of esophageal adenocarcinoma. After esophagectomy, the surgeon elects to perform a supercharged pedicled jejunal flap for complete esophageal reconstruction. Which of the following is the most likely recipient site complication? A) Chylothorax B) Fistula C) Flap failure D) Hematoma E) Isolated neck cellulitis

B All of the complications listed in the options have been described in relationship to esophageal reconstruction with a jejunal flap. The most common complication involves abdominal wound infection, occurring in 21% of patients. The most common medical complication is pneumonia, occurring in 18% of the patients. Regarding the recipient site, in multiple reviews of esophageal reconstruction with jejunal flaps, fistulas have the highest complication rate (up to 14%). The rate of complication for neck cellulitis, hematoma, flap failure, and chylothorax are 5%, 4%, 1%, and 1%, respectively.

A 14-year-old girl is brought to the office because of a 1-month history of a painful, growing lesion in the hard palate with "electric-shock sensations" on palpation. Results of incisional biopsy show adenoid cystic carcinoma of the minor salivary glands. Which of the following is the most appropriate next step in management of this patient? A) Chemotherapy B) CT scan and MRI C) Excision with 1-cm margins D) Excision with 2-cm margins E) Radiation therapy

B Although minor salivary gland tumors are much less common than major salivary gland tumors, minor salivary gland tumors are much more likely to be malignant. Additionally, pediatric salivary cancers represent only about 5% of all salivary cancers, but are also more likely to be malignant (almost 50% were malignant in Armed Forces Institute of Pathology series of 168 pediatric salivary gland tumors). Finally, the palate is the most common source of minor salivary gland tumors, which are more likely to be malignant and higher stage when detected. The clinical presentation of paresthesias of adenoid cystic carcinoma (ACC) suggests perineural invasion. One series (University of Maryland) of 243 minor salivary gland tumors found 78% of them were malignant, and of those malignant tumors, 15% were ACC. Given the perineural invasion symptoms, imaging, in particular MRI, can detect perineural invasion and help plan the degree of surgery. In this scenario, clinical exam pointed to perineural invasion, which should be imaged to plan for surgery. Chemotherapy is not used in the treatment of this disease. Radiation therapy alone is not usually performed, as this is considered a surgical disease. However, it can be used as adjuvant therapy in addition to surgery. Excisional biopsy usually recommends 1- to 2-cm margins. Patients with high-stage, perineural invasion, lymphadenopathy, or other signs of extensive disease may receive surgery with adjuvant radiation therapy. Regardless, ordering a CT scan and MRI is a reasonable initial approach before surgical treatment.

Which of the following is characteristic of a Merkel cell tumor of the head and neck? A ) Arise from minor salivary glands B ) Higher mortality than malignant melanoma C ) Rarely metastasize D ) Treated with radiation therapy alone E ) Typically involve intraoral structures

B Merkel cell tumors are rare neuroendocrine tumors that are highly aggressive. Even despite wide surgical resections, up to one third of patients with Merkel cell tumors have local recurrences. The 5-year mortality rate of Merkel cell tumors is approximately 33%. This is more than double the mortality rate of malignant melanoma (15%). Merkel cell tumors are best treated with wide local excision and sentinel lymph node biopsy. Adjuvant radiation therapy is also used; however, radiation alone is not considered to be the best treatment. Finally, Merkel cell tumors usually arise in sun-damaged skin of the head and neck, not intraoral structures.

A 54-year-old man with a nodule in the soft palate undergoes evaluation with CT scan and biopsy. Results show a malignant oropharyngeal tumor. In order to stage the cancer and determine treatment, which of the following must be performed? A) Dental evaluation via panoramic x-ray study (Panorex) B) p16 status via immunohistochemistry C) Perineural invasion D) Smoking status via urine cotinine E) Speech evaluation via video nasoendoscopy

B Oropharyngeal cancer affects the base of the tongue, soft palate, tonsils, and posterior pharyngeal wall. The most recent National Comprehensive Cancer Network (NCCN) staging guidelines require HPV status to determine staging. HPV status is determined by p16 status via immunohistochemistry on the biopsy sample. The other answers are incorrect because perineural invasion is an important prognostic factor, but is not included in current staging criteria; smoking is related to cancer, but its status is not required for cancer staging; and dental evaluation and speech evaluation are clinically indicated if needed and are not for cancer staging.

A 10-year-old boy is evaluated because of a 6-month history of a slow-growing mass of the left cheek that is soft, nonfixed, and measures 4 cm in diameter. There is no associated adenopathy. Fine-needle aspiration confirms the diagnosis of pleomorphic adenoma; it is anterior/outside of the parotid gland. After complete excision, the parents should be counseled that the recurrence rate is most likely which of the following? A) 1% B) 10% C) 20% D) 40%

B Pleomorphic adenoma is an uncommon lesion, and when it does occur, it is usually in the parotid gland. It does occur in the submandibular, sublingual, and other minor salivary gland locations in decreasing order of frequency. Pleomorphic adenoma is a benign lesion, but the recurrence rate is 6 to 15%. Given the location, wide margins are ideal, but care must be taken to spare the branches of the facial nerve.

A 60-year-old woman undergoes left hemiglossectomy for squamous cell carcinoma of the oral tongue. Reconstruction with which of the following is most likely to achieve the best functional outcome? A) Pectoralis major myocutaneous pedicled flap B) Radial forearm fasciocutaneous free flap C) Rectus abdominis myocutaneous free flap D) Split-thickness skin graft E) Temporalis muscle pedicled flap

B Reconstructive strategies for partial and hemiglossectomy reconstruction are different than for subtotal and total glossectomy reconstruction. For partial and hemiglossectomies, the ideal reconstruction preserves the mobility of the remaining tongue, maintaining its ability to manipulate foods, articulate words, and sweep the oral cavity clean. Such defects are best reconstructed with a thin, pliable flap that resists contraction during the healing process. Of the choices listed, the radial forearm fasciocutaneous free flap best fits those requirements. The pectoralis major myocutaneous pedicled flap is usually too thick to permit ideal movement and can have problems with reach and tethering to the chest as the proximal flap and pedicle contact during healing. The temporalis muscle pedicled flap is unlikely to reach the entire defect and the muscle is likely to contract and stiffen with time, limiting tongue movement. The rectus abdominis myocutaneous free flap is usually too thick to permit unrestricted movement of the remaining hemitongue and is better suited to subtotal and total glossectomy reconstruction, which is best effected with a bulky flap that diverts food and liquids laterally into the pharynx rather than into the larynx. A split-thickness skin graft, while thin, usually contracts substantially, limiting tongue movement. Also, if there is communication into the neck following tongue resection with neck dissection, the skin graft would be at high risk for resulting in a fistula.

An otherwise healthy 35-year-old woman is evaluated for a 3-cm left parotid mass. Physical examination shows weakness of facial muscles on the side of the tumor. CT scan shows several enlarged cervical lymph nodes. Parotidectomy is performed, and pathologic examination shows a mixed population of poorly differentiated epithelial cells and intermediate cells with occasional secretory cells and neural invasion. Which of the following is the most likely diagnosis? A) Hemangioma B) Mucoepidermoid carcinoma C) Pleomorphic adenoma D) Squamous cell carcinoma E) Warthin tumor

B Salivary gland tumors are relatively rare and make up about 3 to 4% of all head and neck neoplasms. Approximately 80% of salivary gland tumors originate in the parotid gland. Approximately 80% of parotid gland tumors are benign. Facial paralysis may be associated with malignant tumors and is a sign of neural invasion. Malignant tumors may also metastasize to the regional lymph nodes and to distant sites. Mucoepidermoid carcinoma is the most common malignancy of the parotid gland and the second most common malignancy of the submandibular and minor salivary glands. Mucoepidermoid carcinomas contain two major elements: mucus-secreting cells, and epithelial cells of the epidermoid variety. Low-grade tumors are associated with a predominance of mucus-secreting cells lining cysts and intervening nests of well-differentiated epidermoid cells. High-grade tumors show few or no mucus-secreting cells and the epidermoid cells are poorly differentiated. Intermediate-grade tumors are defined by less cyst formation than low-grade tumors with nests of epidermoid and less differentiated intermediate cells. The biologic behavior of mucoepidermoid carcinomas correlates with their histologic grade. On the basis of the nerve invasion, the appropriate treatment for this patient includes radical parotidectomy with facial nerve sacrifice. A neck dissection should also be performed for high-grade lesions or those with suspicious adenopathy. Postoperative radiation therapy is usually recommended for higher-grade mucoepidermoid cancers. Pleomorphic adenoma, also known as benign mixed tumor, is the most common benign tumor of the parotid gland. This tumor is histologically characterized by epithelial and connective tissue elements, with stellate and spindle cells interspersed with a myxoid background. Warthin tumor (papillary cystadenoma lymph

A 55-year-old man is referred because of a 1-year history of ear and throat pain. Physical examination shows a 1-cm exophytic tumor of the anterior tonsillar pillar within the oropharynx. Biopsy of the tumor shows squamous cell carcinoma. Which of the following cervical lymphatic levels is most likely to be first involved in this patient? A) I B) II C) III D) IV E) V

B The anterior tonsillar pillar (palatoglossal arch) and tonsil are the most common site for primary neoplasms of the oropharynx. A 1-cm tumor (T1) at this location has a 71% incidence of cervical lymph node metastases. Oropharyngeal tumors arising at the base of the tongue have a similar incidence of lymphatic metastases, whereas oropharyngeal wall and soft palate T1 tumors only metastasize in 8 to 25% of cases. The most direct path of lymphatic drainage from the oropharynx is to level II (jugulodigastric) lymph nodes, which can be examined clinically. From level II the progression is sequential to levels III, IV, and V. It is rare to encounter a "skipped" level. The other less frequent lymphatic drainage pathways detectable only on imaging studies are to retropharyngeal and parapharyngeal nodes. Midline tumors can drain to bilateral lymphatic systems.

A 68-year-old man with a history of laryngeal cancer treated with chemoradiation 2 years ago has a recurrence. He is scheduled for total laryngopharyngectomy with circumferential resection of the pharynx extending from the floor of the mouth to 2 cm above the manubrium. Which of the following is the most appropriate single-stage reconstruction? A) Construction of a spit fistula B) Coverage with an anterolateral thigh flap C) Coverage with a deltopectoral flap D) Coverage with a pectoralis flap E) Use of gastric pull-up

B The circumferential defect described in this patient requires coverage with a tubularized flap that can span the length of the defect and reestablish continuity of the alimentary track. Gastric pull-up is not a good option in this case because of its high morbidity and poor perfusion in the most proximal region of the gastric flap. Coverage with the pectoralis flap or deltopectoral flap is not an appropriate option because these flaps cannot be tubularized in a single-stage reconstruction. The spit fistula would not restore alimentary tract continuity, and it should only be used if no other reconstructive options are available or if the patient is medically unstable. The anterolateral thigh flap is the best option in this case because it can be tubularized to span the defect. In most cases, the resulting reconstruction is highly effective with restoration of swallowing function in the majority of patients.

A 64-year-old man with peripheral vascular disease is evaluated because of an oral squamous cell carcinoma that is invading the mandible. On examination, pedal pulses are not palpable. Surgical resection and postoperative radiation therapy are planned. The resulting defect is expected to extend from the left mandibular parasymphysis to the right mandibular mid body. A flap supplied by which of the following arteries is most appropriate for reconstruction? A) Anterior tibial B) Circumflex scapular C) Superficial circumflex iliac D) Supraclavicular E) Thoracoacromial

B The circumflex scapular artery is the blood supply to the scapular bone or osteocutaneous free flap. The pedicle can be extended by including the subscapular artery and vein proximally. This flap can be designed as a chimeric flap to include the latissimus dorsi and serratus anterior muscles based on the thoracodorsal artery, another branch of the subscapular system, to reconstruct complex defects that involve multiple tissue types. The scapular flap can also be based on the angular branch of the thoracodorsal artery. One disadvantage of the scapular flap is that it is on the back, usually precluding simultaneous oncologic resection and flap harvest to save operative time. Composite bony reconstruction is indicated for the expected mandibular defect. Soft-tissue reconstruction alone for anterior mandibular defects is associated with significant cosmetic deformity as well as impaired masticatory, speech, and even swallowing function. The fibula osteocutaneous free flap, based on the peroneal artery, not the anterior tibial artery, is favored by many surgeons based on its generous bone length and good quality bone stock. However, the flap is contraindicated when blood supply to the distal lower extremity is compromised, such as in advanced peripheral vascular disease (also known as peripheral arterial disease, or PAD). The thoracoacromial artery is the blood supply for the pectoralis major muscle or myocutaneous pedicled flap, which is a soft-tissue flap that would not satisfactorily restore this patient's appearance and function. An osteomyocutaneous variant of the pectoralis major flap that incorporates the fifth rib has been described, but would not be a first-line option due to limited reach of the flap and the tendency to tether the reconstructed jaw to the neck as scar contracture occurs. The superficial circu

A 47-year-old Caucasian man comes to the office regarding a painful enlarging mass at the base of the tongue. He does not smoke cigarettes. The lesion measures 4.5 cm. A biopsy of the lesion is performed and shows (+) p16 staining, nonkeratinized squamous cell carcinoma. Further imaging and workup demonstrate an ipsilateral solitary lymph node measuring 2.3 cm. No distal metastatic disease is found. Which of the following best describes the stage of his disease? A) Stage 1 B) Stage 2 C) Stage 3 D) Stage 4

B The correct answer is Stage 2. Previously, this patient would have been Stage 3. The American Joint Committee on Cancer (AJCC) revised its staging system for squamous cell cancers that stain p16 positive. These lesions are related to the human papillomavirus (HPV) and have been found to be less virulent tongue base or oropharyngeal cancers. Recent studies have demonstrated that 5 year survival difference for patients with Stage 4 disease as <50% for HPV-negative patients and >70% for HPV-positive patients, thus prompting the AJCC to study and revise the staging system for HPV-positive oropharyngeal cancers. These lesions tend to be more sensitive to radiation therapy and chemoradiation and a better prognosis overall. Patients with HPV-related squamous cell cancers tend to be younger, male, and Caucasian. HPV-related squamous cell cancers now represent the majority of newly diagnosed oropharyngeal carcinomas in the United States. This new staging system for HPV (+) related cancers went into effect 1/1/2017.

A 62-year-old woman is evaluated for lower blepharoplasty. On examination, negative vector is noted. Postoperatively, this patient is at increased risk for which of the following conditions? A) Dystopia B) Ectropion C) Enophthalmos D) Lagophthalmos E) Proptosis

B The finding of a negative vector places the patient at an elevated risk for lower lid malposition and ectropion. The negative vector refers to the anatomic relationship on lateral view of the maximum projecting point of the globe and the maximum projecting point of the infraorbital malar prominence. If the globe projects less than the malar prominence, a negative vector exists. Conversely, if the malar prominence projects more than the globe, a positive vector exists. The negative vector finding indicates potentially deficient globe and lid support based on skeletal anatomy. Such patients will often have minor scleral show or lateral lid lag. It is important to recognize these findings prior to blepharoplasty surgery in order to surgically address the risks of ectropion via primary lid suspension during the blepharoplasty. Occasionally, lower lid blepharoplasty may be avoided if a negative vector is present and other conditions such as dry eye exist. Other findings or conditions that are associated with postoperative ectropion and lower lid malposition are: orbicular weakness, anterior lamellar shortage, inferior eyelid/orbital volume deficit, and eyelid laxity. Excessive or prominent middle lamellar scarring can occur after surgery, which can also lead to lid malposition. Lagophthalmos is the inability to lower the upper lid fully and is a negative consequence of upper blepharoplasty due to excessive tissue resection or fibrosis. Enophthalmos is interior retraction or displacement of the globe related to increased orbital volume. This is unrelated to lower blepharoplasty surgery. Proptosis is an external displacement of the globe giving the appearance that the eyeball is extruding from the obit. This is most often associated with Graves disease, head trauma, and increased intracranial pressure. It can also be due t

A newborn has a pretragal cystic mass with a sinus tract visible in the external auditory meatus. Which of the following is the most likely diagnosis? A) Dermoid cyst B) First branchial cleft cyst C) Second branchial cleft cyst D) Third branchial cleft cyst E) Vascular cystic lesion

B The first branchial cleft develops into the external auditory canal. The second, third, and fourth branchial clefts merge to form the sinus of His, which will normally become involuted. When a branchial cleft is not properly involuted, a branchial cleft cyst forms. Occasionally, both the branchial pouch and branchial cleft fail to become involuted, and a complete fistula forms between the pharynx and skin. First branchial cleft cysts are divided into type I and type II. Type I cysts are located near the external auditory canal. Most commonly, they are inferior and posterior to the tragus (base of the ear), but they may also be in the parotid gland. Type II cysts appear at the angle of the mandible and may involve the submandibular gland. The second branchial cleft accounts for 95% of branchial anomalies, and they are most frequently identified along the anterior border of the upper third of the sternocleidomastoid muscle and adjacent to the muscle. However, these cysts may present anywhere along the course of a second branchial fistula, which proceeds from the skin of the lateral neck, between the internal and external carotid arteries, and into the palatine tonsil. Therefore, a second branchial cleft cyst is part of the differential diagnosis of a parapharyngeal mass. Third branchial cleft cysts are rare. A third branchial fistula extends from the same skin location as a second branchial fistula (recall that the clefts merge during development); however, a third branchial fistula courses posterior to the carotid arteries and pierces the thyrohyoid membrane to enter the larynx, terminating on the lateral aspect of the pyriform sinus. Third branchial cleft cysts occur anywhere along that course (eg, inside the larynx), but they are characteristically located deep to the sternocleidomastoid muscle. Congenital dermoid cysts of the face typically occur at the lateral orbit overlying the ZF suture, and vascular cyst lesions can be located anywhere on the face but do not present with sinus tracts into the EAC.

A 24-month-old infant is evaluated because of a mass on the lower lip that her parents first noticed 1 month ago. Physical examination shows a mucosal lesion of the right lower lip that is round, clear, translucent, soft, and measures approximately 0.5 cm in diameter. Which of the following is the most appropriate next step in management? A) CT scan of the face without contrast B) Excisional biopsy C) Fine-needle aspiration D) MRI of the face with contrast E) Ultrasonography of the lesion

B The lesion described is a mucocele, which is a type of cyst likely caused by minor salivary gland mucin seepage. They are most frequently on the lower lip mucosa away from the midline, but they can present anywhere there is oral mucosa. A minority of these resolve on their own, therefore, the majority of the time an excisional biopsy is recommended. Imaging is not indicated for this type of lesion. Fine-needle aspiration is not indicated.

A 57-year-old man undergoes resection of a malignant lesion involving the mandible. He does not smoke cigarettes. Which of the following flaps is most appropriate to use for reconstruction of the resulting mandibular defect from ramus to ramus? A) Cadaveric bone graft with scapular free flap B) Fibular free flap C) Lateral thigh flap with rib graft D) Myocutaneous pectoralis major flap with reconstructive plate E) Osteocutaneous radial forearm free flap

B The most appropriate flap for a large defect from ramus to ramus is a fibular free flap. It allows a large segment of vascularized bone and a skin paddle to be used for reconstruction.The osteocutaneous radial forearm free flap is good for limited osteocutaneous defects, and has less bone available for reconstructing the mandibular defect than the fibular flap. It is not the best choice in this instance, because the bone defect is much larger than this flap can reliably provide.The myocutaneous pectoralis flap has been used historically in mandibular reconstruction; however, it is prone to break down over the reconstruction plate, and is a less ideal choice than a free fibular flap to reconstruct the described defect.A scapular free flap requires repositioning the patient and would not provide adequate vascularized bone to reconstruct the described mandibular defect. It is not typically used, even with cadaveric bone, to reconstruct large mandibular defects.The anterolateral thigh flap is also used in soft tissue reconstruction, but does not provide vascularized bone for reconstructing the large bony defect in this particular case, and non-vascularized rib graft would not be a good substitute for a large mandibular defect.

Which of the following is the primary treatment for keratocystic odontogenic tumor? A) Curettage only B) Enucleation and chemoablation C) Enucleation only D) Marginal mandibulectomy E) Segmental mandibulectomy

B The most common benign tumors of the jaw are ameloblastoma (37%) and keratocystic odontogenic tumor (KCOT) (14%). Ameloblastomas are slow growing, occur in the 4th to 5th decades of life, and arise from odontogenic epithelium. KCOTs are locally aggressive, occur earlier in life, and also arise from odontogenic epithelium. Curettage or enucleation results in higher recurrence rates. Addition of Carnoy's solution (absolute alcohol, chloroform, glacial acetic acid, and ferric chloride) to the enucleated site for 3 minutes addresses the most common issue of local recurrence. Segmental or marginal resection of the mandible is reserved for recurrence after resection locally.

A 65-year-old man is evaluated because of hypernasal speech and nasal regurgitation 12 months after he underwent resection of a soft palate tumor. Examination shows patent nasal passages and no soft palate. Which of the following is the most appropriate treatment? A) Injection of corticosteroids to the inferior turbinates B) Placement of an obturator prosthesis C) Skin grafting D) Tracheotomy E) Observation only

B The patient describes velopharyngeal insufficiency post-soft palate resection. Prosthetic obturation is the traditional means of reconstructing palatal defects. After 12 months, observation has already been performed without resolution of symptoms. Skin grafting of soft palatal defects is of little use because it does not adequately reestablish the bulk necessary for through-and-through palatal defects, therefore leading to contracture and palatal dysfunction. Tracheotomy would exacerbate speech and swallowing difficulties and therefore is not appropriate. Injecting the inferior turbinates with corticosteroids would not functionally correct the velopharyngeal insufficiency and is therefore not appropriate.

The stylopharyngeus muscle is innervated by which of the following nerves? A) Facial (VII) B) Glossopharyngeal (IX) C) Hypoglossal (XII) D) Trigeminal (V) E) Vagus (X)

B The pharyngeal muscles are all innervated by the vagus (X) nerve, except the stylopharyngeus muscle, which is innervated by the glossopharyngeal nerve (IX). The trigeminal nerve (V) is responsible for facial and oral sensation. The maxillary branch (V2) is responsible for sensation of the upper teeth, upper lip, hard palate, cheeks, and nasopharyngeal mucosa. The mandibular branch (V2) provides sensory fibers for the lower teeth, lower mucosa of the mouth and the anterior two-thirds of the tongue. The facial nerve (VII) provides motor innervation of the muscles of facial expression and the posterior bellies of the stylohyoid and digastric muscles. The vagus nerve (X) provides motor innervation to all of the pharyngeal muscles except the stylopharyngeus muscle. The hypoglossal nerve (XII) provides motor innervation to the intrinsic and extrinsic tongue muscles and also provides motor innervation to the geniohyoid muscle through the ansa cervicalis.

A 50-year-old man presents with a 3-cm ulcerated oropharyngeal mass that has appeared and gradually enlarged over the past 4 months. Physical examination is otherwise unremarkable. CT scans of the neck and chest and liver function testing are within normal limits. In addition to biopsy of the mass, which of the following tests is most appropriate for this patient? A) CT angiography of the head and neck B) P-16 protein expression test C) Sentinel lymph node biopsy D) Serum IgA levels for Epstein-Barr virus E) Urine cotinine assay

B This patient's lesion is concerning for oropharyngeal squamous cell carcinoma (SCC). To establish a diagnosis and direct treatment, fine-needle aspiration biopsy of the mass should be the next step in this patient's workup. In addition to histologic examination, the specimen should be tested for p16 expression, which is a marker for HPV-associated cancer. HPV can infect the oral cavity and pharynx and cause oropharyngeal and, more rarely, oral SCC. HPV-positivity has important prognostic value and is staged differently than non-HPV-positive SCC's. HPV-associated oropharyngeal SCC has a relatively indolent course and a better prognosis than non-HPV-associated oropharyngeal SCC. Treatment for early stage disease may either be radiation with or without chemotherapy and/or immunotherapy or surgery, including transoral robotic surgery. Survival and recurrence rates are similar for the two treatment strategies. The HPV vaccine protects against the types of HPV that cause oropharyngeal cancers. Epstein-Barr virus (EBV) is implicated in the development of nasopharyngeal carcinoma (as well as certain types of lymphoma), not oropharyngeal cancer. While not used for diagnosis, measurement of EBV DNA may be done before and after treatment to monitor response. Oropharyngeal cancers are not especially vascular, unlike juvenile nasopharyngeal angioma, which occurs in the nasopharynx and nasal cavity, not the oropharynx, so CT angiography is not indicated. Sentinel lymph node biopsies are used to assess for occult lymph node metastases in the clinically N0 neck. Their use in squamous cell cancer is controversial and would not be the next step in making the diagnosis. Urine cotinine assay is a test for cotinine, a byproduct of nicotine used to evaluate exposure to tobacco. While tobacco is an important risk factor for head and neck cancer, its use is not helpful in making a diagnosis or directing treatment.

A 20-year-old man is brought to the emergency department after sustaining a stab wound to the neck during a violent assault. Physical examination shows an expanding neck hematoma and stridor. Intraoperative exploration shows a deep laceration to the anterior lateral neck at the level of thyroid cartilage and profuse extravasation of blood from the carotid sheath. According to anatomical zone-based classification of penetrating neck injuries, which of the following zones is involved? A) Zone 1 B) Zone 2 C) Zone 3 D) Zone 4

B Zone 1 extends from clavicles to cricoid, zone 2 from cricoid to angle of mandible, and zone 3 from angle of mandible to skull base.There are only 3 zones in penetrating neck injuries.

A 45-year-old man presents with a rapidly growing, painful mass of the left cheek. He has facial weakness on the left side. CT scan shows a left parotid tumor encasing the facial nerve; there are no abnormal lymph nodes. Which of the following is the most appropriate surgical treatment for this lesion? A) Enucleation B) Partial parotidectomy C) Radical parotidectomy D) Superficial parotidectomy E) Total parotidectomy sparing the facial nerve

C Although most (80%) of parotid tumors are benign, the parotid gland is the most common location for a salivary gland malignancy. Pain, paresthesia, and facial paralysis are signs of neural invasion and are usually associated with malignant tumors. Rapid growth, bony fixation, skin ulceration, and palpable nodal enlargement are also associated with malignancies. Parotidectomy is removal of part or all of the parotid gland. While not true "lobes," the facial nerve anatomically divides the parotid gland into superficial and deep portions. A superficial parotidectomy involves removal of the parotid gland superficial to the plane of the facial nerve and is appropriate for benign and malignant tumors confined to this portion of the gland. A less than complete superficial parotidectomy that still removes the entire tumor with a negative margin is referred to as a partial parotidectomy. A total parotidectomy involves removal of both the superficial and deep lobes. The facial nerve is carefully dissected and spared if it is not involved. Enucleation, which involves simple removal of the mass, is not indicated for malignant tumors and felt to be controversial for benign tumors, with many surgeons advocating against it. For those that consider enucleation, resection should include the capsule of the tumor, and it should be reserved for benign superficial tumors less than 4 cm in diameter. A radical parotidectomy is indicated in this case of a malignant lesion invading the facial nerve. This procedure involves total parotidectomy with facial nerve sacrifice. If adjacent structures such as the skin, mandible, or temporal bone are involved, an extended radical parotidectomy may be indicated. In the clinically and radiographically node-negative neck, the decision to perform a neck dissection or treat the neck with adjuvant radiation versus observation usually depends on histologic factors and/or tumor subtype and grade.

Compared with oropharyngeal squamous cell carcinomas associated with tobacco and alcohol, which of the following is true about oropharyngeal squamous cell carcinomas associated with human papillomavirus (HPV)? A) They are associated with active HPV infection in the partner B) They are more resistant to radiation therapy C) They have a better prognosis, stage for stage D) They more frequently occur in the hypopharynx E) They occur more frequently in women

C An estimated 53,260 cases of head and neck squamous cell carcinomas (HNSCCs) were anticipated in 2020 in the United States. 70% of the cases (38,380) will be in men. More cases in both women and men will be associated with human papillomavirus (HPV), and it is anticipated that HNSCCs associated with HPV will soon outnumber cases of HPV-associated cervical cancer.Interestingly enough, partners of patients with HPV-associated HNSCCs screened for the HPV16 subtype have the same occurrence rate as the general population (1.2 to 1.3%). HPV-associated HNSCCs are associated with a greater number of lifetime sexual partners (N ? 9) and a greater number of partners involved with orogenital sex (N ≥ 4).HPV-associated HNSCCs typically occur in the oropharynx, whereas lesions on the larynx and hypopharynx are associated with the larger field affected by tobacco smoke and alcohol.Stage for stage, the prognosis is better for HPV-associated HNSCCs. To prevent over-treatment (or to encourage less aggressive treatment), the eighth edition of the Cancer Staging Manual of the American Joint Committee on Cancer has revised the tumor, node, metastasis (TNM) classification of HNSCCs to differentiate between HPV-positive and HPV-negative tumors.

Which of the following structures is formed from the same branchial arch as the vagus (X) nerve? A) Inferior parathyroid glands B) Lesser horn of hyoid C) Levator veli palatini D) Maxillary artery E) Styloid process

C Each of the six branchial arches gives rise to a branch of the aortic arch, a cranial nerve, muscular structures, and skeletal structures. The fourth branchial arch gives rise to the right proximal subclavian artery, the aortic arch, the vagus (X) nerve, the superior laryngeal nerve, the intrinsic muscles of the levator veli palatini, cricothyroid muscles, laryngeal cartilages, and the superior parathyroid glands.The styloid process is derived from the second branchial arch, along with the stapedial and hyoid arteries, the facial (VII) nerve, the muscles of facial expression, the stapes, the lesser horn of the hyoid bone, and the crypts of the palatine tonsils.The maxillary artery is derived from the first branchial arch, along with the trigeminal (V) nerve, the muscles of mastication, anterior belly of the digastric muscle, tensor tympani, tensor veli palatini, mylohyoid, mandible, incus and malleus, maxilla, vomer, zygoma, and temporal bone.The inferior parathyroid glands are derived from the third branchial arch, along with the common carotid artery, internal carotid artery, glossopharyngeal (IX) nerve, stylopharyngeus muscle, greater horn of the hyoid bone, and thymus.The lesser horn of the hyoid bone is derived from the second branchial arch.

A 75-year-old man is scheduled to undergo a 5-cm composite resection including selective neck dissection and adjuvant radiation therapy because of a floor-of-mouth squamous cell carcinoma that is invading the anterior mandible. History includes hypertension. He had smoked a pack of cigarettes daily for 50 years and quit 8 years ago. He has no other cardiac risk factors and walks a mile daily. Which of the following is the most appropriate reconstruction to maximize this patient?s postoperative function? A ) Iliac crest bone grafting B ) Reconstruction with a 2.0-mm titanium plate and coverage with a pectoralis major musculocutaneous pedicled flap C ) Reconstruction with a fibula osteocutaneous free flap D ) Reconstruction with a radial forearm fasciocutaneous free flap E ) Reconstruction with a supraclavicular island pedicled flap

C Failure to reconstruct the anterior mandibular bone results in the so-called ?Andy Gump? defect and is associated with disfigurement, as well as impaired speech, swallowing, and mastication. Of the choices listed, the fibula osteocutaneous free flap is the most reliable reconstruction of the anterior mandible, particularly when radiation therapy is administered. The fibula provides a large amount of good-quality bone stock than can tolerate osteotomies, which are needed to restore the shape of the anterior mandible. A skin paddle based on perforating vessels from the peroneal artery can be included to simultaneously reconstruct the floor-of-mouth mucosal defect. Age alone is not a contraindication to reconstruction with a microvascular free flap, provided the patient is in otherwise good medical condition and has patent blood circulation to his distal lower extremity. A nonvascularized iliac crest bone graft can be used to reconstruct small mandibular defects (under 5 cm in length), but will most likely not tolerate radiation, so it would not be a good option in this case. Mandibular reconstruction with a titanium plate, alone or covered with a soft-tissue flap, such as the pectoralis major myocutaneous pedicled flap, is associated with a very high rate of complications, including infection, exposure, and plate fracture, particularly in the setting of an anterior location and radiation therapy. The radial forearm fasciocutaneous free flap can be used to close the floor-of-mouth mucosal defect, but will not restore the bony mandible. The supraclavicular island pedicled flap is based on the supraclavicular artery, a branch of the transverse cervical artery, and can be used to close oral soft-tissue defects, but, like the radial forearm fasciocutaneous free flap, will not restore the mandible.

A 57-year-old man undergoes superficial parotidectomy. Facial nerve neuropraxia results in gustatory sweating and which of the following additional symptoms? A) Anosmia B) Base of tongue dysgeusia C) Hyperlacrimation D) Migraine headache E) Synkinesis

C Hyperlacrimation, or Bogorad syndrome, is a known complication after Bell palsy or other injury and insults to the facial nerve. Similar to Frey syndrome, the predominant theory for this form of gustatory hyperlacrimation is due to aberrant facial nerve regeneration. Epiphora in general can also occur due to poor "pumping mechanisms" in the eyelids as well as prolonged ectropion and conjunctival show after facial nerve injury. However, hyperlacrimation during gustatory activity is a specific and definable pathology. Treatment for this syndrome includes subtotal lacrimal gland resection, botulinum toxin type A, and various forms of enlarging the lacrimal tract. Synkinesis is a common event after facial nerve regeneration, when the nerve improperly fires and there is lack of typical mimetic muscle coordination. The anterior portion of the tongue taste buds are innervated by facial nerve fibers from the chorda tympani to the lingual nerve, but the base of the tongue is innervated by cranial nerves IX and X. Anosmia is loss of smell that occurs through cranial nerve I injury or obstruction and can lead to taste disturbances. Migraine headaches can be associated with a variety of syndromes and need to be differentiated from other forms of headaches. Ramsay Hunt syndrome can lead to facial nerve dysfunction and facial pain, but this pain is not associated with facial nerve regeneration or migraine headaches.

Which of the following types of head and neck tumors are most often associated with Epstein-Barr virus infection? A) Larynx B) Maxillary sinus C) Nasopharynx D) Tongue E) Tonsil

C Nasopharyngeal cancers are most often associated with Epstein-Barr virus (EBV) infections and arise from the mucous epithelium of the nasopharynx and are relatively rare in the United States. However, these tumors are endemic in Africa and East Asia, accounting for as many as 18% of head and neck cancers in China. Nasopharyngeal tumors are classified as either squamous cell cancers, keratinizing undifferentiated carcinoma, or non-keratinizing undifferentiated carcinoma. EBV infection is most strongly associated with the non-keratinizing undifferentiated subtype and is thought to increase malignant transformation. Nasopharyngeal cancers are most commonly treated with chemotherapy and radiation, with surgery reserved for recurrent or unusual cancers. Reconstruction of skull base defects is most commonly performed using microsurgical transfer of soft-tissue flaps. Alcohol and tobacco are the most common risk factors for head and neck cancers in general, and laryngeal cancers in particular, with cigarette smoking increasing the lifetime risk 5- to 25-fold. Other risk factors for head and neck cancers in general include cigar smoking, environmental exposures, dietary factors (red meat, betel nuts), and human papillomavirus (HPV) infection. HPV infections are most commonly associated with oropharyngeal cancers (tongue, tonsil). Significant risk factors for maxillary sinus cancers include cigarette smoking and environmental factors such as exposure to wood dust.

A 70-year-old man is evaluated following tumor resection. Physical examination shows a 4 × 4-cm defect of the right maxilla that includes all of the teeth posterior to the right canine (two premolars and three molars) but spares the right orbital floor. He did not undergo radiation therapy. He wishes to restore mastication, speech, and swallowing by the simplest means that will still be efficacious. Which of the following is the most appropriate method of reconstruction? A) Fibula osteocutaneous free flap with osseointegrated implants B) Osseointegrated implant-retained prosthesis C) Prosthetic obturator D) Rectus abdominis musculocutaneous free flap with a conventionally retained dental prosthesis E) Temporalis muscle pedicled flap

C Palatal obturators can adequately restore missing maxillary dentition as well as prevent oronasal leakage of air, liquids, and foods. They have the advantage of being removable, which permits visualization of the maxillary cavity for tumor surveillance. Prosthetic retention can be difficult or impossible in sizable defects, particularly when there are few teeth to stabilize the prosthesis. In this patient who has sufficient remaining maxillary teeth and the majority of the alveolar arch, the prosthesis is expected to have good stability, and would be the appropriate choice in a patient who wishes to avoid further invasive procedures. The temporalis muscle flap can be transposed into the oral cavity and can be used for closing defects of the palate. However, this flap alone would not provide replacement of the missing dentition and is still more invasive than a palatal obturator. Additionally, the temporalis muscle flap results in marked temporal hollowing at the donor site. The rectus abdominis musculocutaneous free flap can close the palatal defect and restore shape to the cheek in patients with a unilateral maxillectomy. In combination with a dental prosthesis, the rectus abdominis musculocutaneous free flap can restore the patient's appearance and function. However, the rectus abdominis musculocutaneous free flap is also an invasive procedure, and it can sometimes be challenging to inset the flap such that there is enough room in the mouth for a prosthesis. In a patient who has had a maxillectomy, there is generally inadequate remaining bone stock to place osseointegrated implants for prosthetic retention. The patient's existing dentition should be adequate to support a prosthesis. The fibula osteocutaneous free flap and other osteocutaneous flaps can be used to close the palatal defect to prevent nasal regur

A 51-year-old male carpenter requires a partial glossectomy for recurrent oral squamous cell carcinoma. He runs for five miles three days a week. He underwent radiation therapy two years previously. Microsurgical transfer of which of the following free flaps is most appropriate for reconstruction? A) Deltopectoral flap B) Rectus abdominis flap C) Sural artery perforator flap D) Vastus lateralis flap

C Partial glossectomy defects require a small, thin, pliable flap for optimal reconstruction. The workhorse for glossectomy reconstruction has long been the radial forearm flap (RFF), which often requires a skin graft for donor site closure. However, various reports of donor site morbidity related to the RFF, including delayed healing, decreased grip and pinch strength, and radial nerve sensory problems, make this flap less suitable for a patient whose vocation involves manual labor. The sural artery perforator flap has become increasingly popular as an alternative donor site for very thin, pliable tissue. This flap, which usually arises from perforators from the medial sural artery, results in minimal donor morbidity. Muscle flaps are less desirable for intraoral reconstruction due to the need for an epithelialized surface and the difficulty in achieving skin graft adherence. The rectus abdominis flap is too bulky for partial glossectomy reconstruction. The deltopectoral flap, which arises from the 1st intercostal perforator, is rarely transferred as a free flap. Sacrifice of the vastus lateralis muscle in an avid runner is not recommended.

In patients with ameloblastoma, which of the following cell populations gives rise to this tumor? A) Cementoblast tissue B) Enamel C) Gingiva D) Nerve root E) Odontogenic epithelium

E Ameloblastomas are benign odontogenic tumors derived from odontogenic epithelium. They are typically slow growing, and present in the fourth or fifth decade of life as a mandibular mass in most individuals (80%). Odontogenic cementoblast tissue is not appropriate because this tissue gives rise to an extremely rare benign odontogenic tumor, the cementoma. Gingiva is the mucosal covering of the alveolar bone. Enamel is the dense compound of teeth.

A 75-year-old man comes to the office because of a squamous cell carcinoma of the lower lip. A wide local excision, removing two thirds of the lower lip extending inferiorly to the chin crease but sparing both oral commissures and bilateral selective neck dissections, is performed. Which of the following is the most appropriate method of reconstruction? A) Estlander flap B) Facial artery musculomucosal flap C) Karapandzic flap D) Primary closure E) Radial forearm fasciocutaneous free flap

C The Karapandzic technique involves performing circumoral incisions, mobilizing the orbicularis oris muscle, and preserving the nerves as well as the vascular supply to the lips from the superior and inferior labial arteries. The advantage of Karapandzic flaps is that they maintain a continuous circle of functioning orbicularis muscle, which helps to restore oral competence. Microstomia can occur, but is usually less of a problem in older patients who have greater tissue laxity. The optimal method for lip reconstruction depends on the size of the defect, the location of the defect, and the laxity of the remaining tissues. Primary closure can lead to excellent cosmetic and functional results but is restricted to wounds less than one third to one half of the lip width. The Estlander flap is a full-thickness lip transposition flap that borrows tissue from the opposite lip and can be used to reconstruct defects up to two thirds of the lip width. However, the Estlander flap is specifically designed to restore defects of the oral commissure. The Abbé flap is also a transposition flap that "steals" tissue from the opposite lip and is used for central defects, but must remain pedicled by a bridge of lip for several weeks, severely restricting mouth opening during that time. The radial forearm fasciocutaneous free flap is used for total lower lip defects, particularly in situations where sufficient cheek tissue cannot be recruited to close the defect (which is known as the Webster-Bernard technique). Suspension of the folded radial forearm flap over a palmaris longus tendon graft secured to the maxillary bone helps prevent flap ptosis that can result in loss of oral competence, a major problem that can occur with this technique. The facial artery musculomucosal (FAMM) flap is a pedicled flap consisting of buccal mucosa a

A 12-year-old boy with a thyroglossal duct cyst undergoes a Sistrunk procedure. Which of the following structures are resected during this procedure? A) Cyst and cyst tract only B) Cyst, cyst tract, and middle third of the cricothyroid cartilage C) Cyst, cyst tract, and middle third of the hyoid bone D) Cyst, cyst tract, and middle third of the thyroid cartilage E) Cyst, cyst tract, and the pyramidal lobe of the thyroid

C The Sistrunk procedure is the operation of choice for thyroglossal duct cysts. This operation involves resection of the cyst, the cyst tract, and the middle third of the hyoid bone. In the Sistrunk procedure, the thyroid cartilage is not removed, nor is the cricothyroid cartilage. If, upon exploration, the distal tract is found to be in communication with the pyramidal lobe of the thyroid, then the communication should be excised. Despite this, resection of the pyramidal lobe of the thyroid is not a standard component of the Sistrunk procedure.

An 8-year-old boy is brought to the clinic for evaluation of a congenital ear deformity characterized by an unfolded antihelix. During the embryological period, which of the following structures failed to develop appropriately? A) First pharyngeal cleft B) First pharyngeal pouch C) Second pharyngeal arch D) Third pharyngeal arch E) Third pharyngeal pouch

C The ear development starts during the third month of gestation from six hillocks that arise on the first and second arches. The first pharyngeal cleft develops into the external auditory meatus. The second through fourth pharyngeal clefts are usually obliterated due to the expansion of the second pharyngeal arch. The first pharyngeal pouch develops into the auditory tube and middle ear cavity. The second through fourth pharyngeal pouches are not related to ear development, nor is the third pharyngeal arch. The second pouch becomes the palatine tonsils' crypts, while the third pouch forms parathyroid glands and thymus. The fourth pouch differentiates into parathyroid glands and parafollicular cells of the thyroid gland. The cartilaginous component of the third pharyngeal arch gives rise to part of the hyoid bone.

A tumor that has metastasized to the mandible in a 70-year-old man is most likely to have originated from which of the following anatomic sites? A) Colon B) Kidney C) Lung D) Testis

C The mandible is an uncommon site for metastatic tumor spread and is usually evidence of more widespread disease. They may arise from numerous sources and usually present with the discovery of a new nodular mass with or without ulceration. Imaging studies demonstrate radiolucent/hypodense lesions. In male patients, the most common age range is the fifth to seventh decades. In men, the most common primary site is the lung; in women, it is the breast. Of note, the posterior mandible appears to be the most common bony site of involvement. The mean survival time is 8 months.

An otherwise healthy 59-year-old woman is scheduled to undergo resection of recurrent squamous cell cancer of the right temporal area 5 years after initial resection, superficial parotidectomy, limited upper cervical lymphadenectomy, skin grafting, and adjuvant radiation therapy. The anticipated defect will be 6 × 8 cm, involving the skin and subcutaneous tissues. The superficial temporal vessels cannot be identified. Which of the following is the most appropriate method of wound closure? A) Lateral arm flap B) Parascapular flap C) Radial forearm free flap D) Scalp rotation flap E) Split-thickness skin grafting

C The most appropriate method for wound closure is a radial forearm free flap, as it matches the thin skin and subcutaneous tissue of the temporal area and has a long pedicle that can reach recipient vessels in the neck. With a patient history of radiation therapy and superficial parotidectomy, the superficial temporal vessels are unlikely to be suitable recipient vessels. With a patient history of upper neck dissection, it is possible that suitable recipient vessels will only be found inferiorly, and thus, it is best to use a flap with a long, reliable pedicle. A split-thickness skin graft will likely fail in a radiated wound bed. A scalp rotation flap will likely include tissue in the radiation field, bring hair-bearing tissue into a non-hair-bearing area, and require a split-thickness skin graft for closure of the donor site. It could be considered in a patient who is not a candidate for free tissue transfer. The lateral arm flap and parascapular flap are slightly thicker than the radial forearm flap, but the principal reason to avoid these flaps in this patient is that their pedicle lengths are relatively short and may not reach the recipient vessels in the neck.

A 45-year-old woman with no history of smoking presents with a 1.4-cm squamous cell carcinoma of the left anterior mandibular gingiva and an upper cervical neck mass. Full-thickness cortical invasion of the tumor into the superior mandible as well as a single enlarged lymph node is observed on CT scan. In addition to neck dissection, which of the following is the most appropriate surgical treatment? A) Marginal mandibulectomy B) Radiation therapy C) Segmental mandibulectomy D) Transoral laser surgery E) Wide local excision with periosteal stripping

C The most appropriate surgical treatment for transcortical invasion of the mandible is a segmental mandibulectomy, in which a full-thickness portion of the mandible is excised. Marginal mandibulectomy is reserved for cases in which the cancer stops at the periosteum or does not penetrate full-thickness through the cortex of the mandible. Wide local excision with or without periosteal stripping, and transoral laser surgery are not adequate treatment for this lesion. Radiation therapy for this advanced stage cancer is also not adequate. Combined surgery followed by postoperative radiation therapy is indicated.

A 25-year-old woman is evaluated because of facial swelling around the jaw and loosening teeth. The swelling has worsened progressively. Physical examination shows unilateral right facial swelling around the third molar. CT scan of the mandible shows a radiolucent, multicystic, unilocular lesion in the right mandibular angle and confirmed root resorption. Which of the following series of treatments is most appropriate for this patient? A) Local curettage of the lesion followed by cancellous bone graft reconstruction B) Neo-adjuvant radiation therapy, segmental mandibulectomy, and reconstruction C) Segmental mandibulectomy and reconstruction D) Segmental mandibulectomy, reconstruction, and postoperative chemotherapy after adjuvant therapy E) Segmental mandibulectomy, reconstruction, dental rehabilitation, and sentinel node biopsy

C The patient described has an ameloblastoma. Ameloblastomas are benign tumors of odontogenic origin. Treatment is surgical. Conservative management, such as local curettage, is associated with high recurrence rate. The most appropriate treatment is segmental mandibulectomy, reconstruction, and dental rehabilitation. Because ameloblastoma is benign, neither adjuvant therapy nor neoadjuvant therapy is indicated.

A 62-year-old woman comes to the office because of skin necrosis and scabbing following a minor injury 6 weeks ago. A photograph is shown. History includes glioblastoma that was treated with craniotomy and radiation therapy 20 years ago. Debridement is performed, exposing the calvaria and dura. Which of the following is the most appropriate treatment for this patient? A) Bone burring with skin grafting B) Coverage with acellular dermis matrix C) Coverage with a free flap D) Coverage with scalp rotation flaps E) Negative pressure wound therapy

C The patient described has severe radionecrosis of the skin and the underlying bone from the previous radiation therapy. The skin is thinned, and spontaneous necrosis with scabbing can be seen in the photograph and will require extensive debridement including necrotic skin and underlying bone. As a result, closure with scalp rotation flaps will be inadequate because of the extensive skin damage, size of the defect, and exposed underlying dura. Similarly, bone burring will not be possible even if the underlying calvaria was preserved due to the history of radiation. Negative pressure wound therapy is not an option due to exposure of the underlying dura and brain matter. Similarly, acellular dermis will not be effective due to the damaged underlying tissues and chronic contamination. Radical debridement and coverage with a free flap (e.g., latissimus flap) will offer the best option for reconstruction in this patient because this procedure will deliver healthy vascularized tissues to cover the exposed cranial structures.

An obese 65-year-old man who undergoes resection of an oral tongue nodule has a 5 × 9-cm defect of the hemitongue and floor of the mouth. Which of the following is the most appropriate method of reconstruction? A) Full-thickness skin grafting B) Primary closure C) Radial forearm fasciocutaneous flap D) Rectus abdominis musculocutaneous flap E) Submental artery island flap

C The radial forearm fasciocutaneous free flap is the most appropriate reconstructive choice among those listed. It is generally a thin, pliable flap with a long pedicle and vessels of adequate caliber for straightforward microvascular anastomosis. The physical characteristics of this flap are well-suited to preserving the mobility of the remaining hemitongue, resulting in reasonable speech and swallowing function following surgery in most cases. The primary goals of reconstruction following hemiglossectomy include watertight wound closure such that oral secretions do not communicate with the neck contents and result in a fistula, and restoration of speech and swallowing function by preserving the mobility of the remaining native tongue. Primary closure would result in severe tongue tethering and impaired speech and swallowing. Although likely to contract less than a split-thickness skin graft, a full-thickness skin graft may also restrict mobility of the tongue and is unlikely to achieve a watertight wound closure. The submental artery island flap is based on a branch of the facial artery that can be used to close defects up to approximately 7 × 18 cm, depending on neck skin laxity. This flap is generally unreliable following neck dissection in which the facial artery and its branches may be ligated. The rectus abdominis musculocutaneous free flap is a highly reliable free flap but is often too bulky to permit optimal mobility of the remaining tongue, especially in patients with truncal obesity.

A 32-year-old man comes to the office because of a pleomorphic adenoma in the right parotid. A mass is palpated over the angle of the mandible. Examination of the oral cavity and neck shows no abnormalities. Which of the following is the most appropriate surgical treatment? A) Enucleation B) Radical parotidectomy C) Superficial parotidectomy D) Total parotidectomy E) Total parotidectomy and neck dissection

C The salivary gland neoplasms are uncommon and generally benign. Most benign tumors can be easily cured by wide local excision. Pleomorphic adenoma is the most common salivary gland tumor, with a propensity for local recurrence. Simple enucleation is discouraged. Removal is usually done by a superficial parotidectomy. The superficial lobe is anatomically defined by the traversing facial nerve. The nerve is preserved, as pleomorphic adenoma is a benign tumor that does not infiltrate the nerve. The parotid gland is also the most frequently affected major salivary gland, and the palatal salivary tissue is the most commonly affected minor salivary gland. Pleomorphic adenomas are the most common neoplasm of salivary glands, comprising 45 to 75% of all tumors in most series. These tumors typically affect patients in their 20s to 50s and there is a female predilection. Warthin's tumor is the second most common tumor of the salivary glands and constitutes approximately 14 to 21% of salivary gland neoplasms. The tumor is almost exclusively found in the parotid gland, typically affects males in their 50s to 60s, and often may be bilateral.

A 50-year-old woman with hypertension and diabetes comes to the office because of a large mid-palatal cancer. She undergoes a total palatectomy. A photograph is shown. Which of the following is the most appropriate method of reconstruction? A) Anterolateral thigh free flap B) Bilateral temporalis muscle flaps C) Fibula osteocutaneous free flap D) Iliac crest bone grafting E) Rehabilitation with a palatal obturator

C This patient has a bilateral maxillectomy defect following resection via a Le Fort I osteotomy. While small defects can be successfully addressed with prosthetic palatal obturators that fit through the wound margins and clasp to the remaining teeth, larger defects can rarely be obturated because of their weight and instability due to lack of dentition and an alveolar ridge. Bone grafting is also not an option for such a large defect, particularly one resulting from a malignancy where postoperative radiation therapy is likely. In general, bone grafts are only indicated in benign conditions with bone loss less than 5 cm in length. Even when these conditions are met, they require coverage with well-vascularized tissue. Temporalis muscle flaps can be used to reconstruct palatal defects, but in this case, the skeletal elements of the mid face have been removed. Temporalis muscle flaps alone will result in loss of mid face projection. Similarly, the anterolateral thigh free and rectus abdominis myocutaneous free flaps are frequently used to reconstruct posterior maxillary defects but, though bulkier than temporalis muscle flaps, will not maintain midfacial projection in this patient with loss of the entire palate and premaxilla. The most appropriate reconstruction for this patient is the fibula osteocutaneous free flap. This flap will restore midfacial height, width, and projection. It has adequate bone stock for osseointegrated implant placement dental restoration as well. The skin paddle is used to close the palatal defect, separating the oral cavity from the nasal cavity.

A 45-year-old man is evaluated because of a 5-cm mass at the angle of the mandible. A CT scan shows an intraparotid mass. Ultrasound-guided fine-needle aspiration shows benign findings. Which of the following is the most appropriate next step in management? A) Chemotherapy and superficial parotidectomy B) Follow-up evaluation in 3 months C) MRI D) Superficial parotidectomy only E) Total parotidectomy

D Most parotid tumors are benign, but they can grow to a large size and produce significant symptoms of discomfort and distortion of anatomy. The differential diagnosis is aided by the use of imaging, CT, or MRI to confirm the location of the mass. Ultrasound-guided fine-needle aspiration is a useful next step in diagnosis of the majority (92%, Sharma, et al). MRI would be superfluous in the present case since imaging is sufficient. Benign neoplasms include pleomorphic adenoma, mucocele, branchial cleft cysts, and lymph nodes. Malignancies include adenocarcinoma.

A 74-year-old man with type 2 diabetes mellitus develops squamous cell carcinoma (SCC) of the mandible. His history is significant for bilateral distal fibular fractures sustained 10 years ago from a motor vehicle collision, which were repaired with open reduction and internal fixation. He states that the ability to eat and chew will be important to him. Which of the following statements regarding mandibular reconstructive options is correct? A) The bone quality of the scapular flap is superior to the iliac crest flap B) Dental rehabilitation with implants is more predictable in osteocutaneous radial forearm flap than the iliac crest flap C) Free fibular flaps have a higher rate of donor site morbidity than iliac crest flaps D) A history of distal fibular fractures is not an absolute contraindication to the use of a free fibular flap E) Iliac crest bone flaps have lower failure rates than osteocutaneous radial forearm flaps

D A history of fibular fracture is not an absolute contraindication to using a free fibular flap. According to a systematic review, iliac crest flaps have the highest rate of failure when compared to all the other methods combined (fibular, radial, scapular). While not contraindicated, dental implants are less frequently placed in patients reconstructed with radial forearm bone flaps. More donor site morbidities have been reported for fibular than iliac crest flaps. Scapular flaps provide inferior bone quality to fibular and iliac crest flaps.

A 65-year-old man presents with facial flushing and sweating with eating 4 weeks after undergoing parotidectomy. Which of the following tests will be most likely to establish the diagnosis? A) Cottle maneuver B) Ice pack test C) Jones test D) Minor iodine-starch test E) Schirmer test

D Auriculotemporal syndrome, or Frey syndrome, can occur after parotidectomy and less commonly after trauma, other facial surgeries (e.g., rhytidectomy), and neck dissection. It results from abnormal innervation (synkinesis) of sweat glands from the postganglionic parasympathetic fibers in the parotid. Symptoms include flushing, sweating, neuralgia, burning, and itching in response to gustatory stimulus. Diagnosis of Frey syndrome is based on clinical history, but the diagnosis can be confirmed with the Minor iodine-starch test: the ipsilateral face is painted with iodine and the patient is challenged with a sialogogue (e.g., lemon juice). Areas of gustatory sweating will turn blue. The Jones test assesses patency of the lacrimal drainage system. The Cottle maneuver assesses the internal nasal valve. The ice pack test is used to assess myasthenic ptosis. The Schirmer test is a tear production test that can be used to assess dry eyes before blepharoplasty.

Early replacement of an avulsed tooth is critical to increasing its chance of long-term survival because it minimizes further damage to which of the following structures? A) Cementum B) Dentin C) Enamel D) Periodontal ligament E) Pulp

D Avulsion of a permanent tooth is one of the most serious dental injuries that may be encountered by a plastic surgeon. Without prompt intervention, the tooth may be lost, committing the patient to a bone graft, an implant, or lifelong prosthodontic appliance. When a permanent tooth is fully avulsed, exposure of the periodontal ligament can lead to its desiccation and cell death within an hour of injury. The long-term viability of a tooth with a dry time of greater than 60 minutes is poor. If the tooth cannot be immediately replanted, it should be stored in a neutral medium such as milk, saliva (in the mouth), or Hanks balanced storage solution. Enamel and cementum make the hard outer layer of the tooth (enamel at the crown and cementum at the root) and are relatively resistant to damage with an avulsion. Dentin is the thick layer deep to the enamel/cementum, which makes up the bulk of the tooth and is not typically injured in a clean avulsion. The pulp contains the neurovascular supply of the tooth. In a complete avulsion, the pulp will become nonviable and will need to be removed with a root canal procedure after the tooth has become stable. However, with appropriate treatment, the tooth can survive after the pulp has been removed.

A 77-year-old man is anticipated to have a 4-cm lateral segmental defect of the mandible after undergoing extirpation of an oral tumor. Reconstruction of the mandibular defect with an iliac crest bone graft is planned. Which of the following should be considered a CONTRAINDICATION to the use of nonvascularized bone graft for mandible reconstruction in this patient? A) Defect greater than 3 cm B) Defect involving the mandibular body C) Extirpation of a malignant lesion D) Need for adjuvant radiation therapy E) Patient age greater than 75 years

D In most cases, vascularized bone flaps provide a reconstruction superior to autologous bone grafts in the treatment of segmental mandibular defects. However, they require more operative time, a longer hospital stay, and specialized surgical teams, and they have the potential for greater donor morbidity. There is evidence that for many cases, bone grafts can be used with acceptable results. Patients undergoing radiotherapy for malignant tumors have a significantly lower success rate with bone grafting than those undergoing vascularized bone flaps. While associated with a need for radiation, a diagnosis of malignancy itself does not predict graft failure. Lateral defects have similar outcomes whether bone grafts or flaps are used, likely due to the lower torsional forces in these regions than at the symphysis. While there is no consensus on the optimal maximal graft length, many consider 6 cm to be the maximum advisable length for a bone graft in this context. Microsurgical free tissue transfer can be successfully accomplished in patients in their 70s and 80s, albeit with a higher rate of complications. Age should not be considered an absolute contraindication in this case.

A 23-year-old African-American man presents with a raised thickened scar on his anterior chest that he complains is pruritic and unattractive. It was removed by another provider 4 years earlier and has slowly recurred over the past year. On examination, the lesion extends beyond the initial borders of the scar and is firm and hyper-pigmented. On review of his prior pathology report, which of the following histologic characteristics is most likely? A) Greater ratio of type III to type I collagen B) Multitude of myofibroblasts and smooth muscle actin C) Parallel collagen bundles D) Thick, wavy, and randomly oriented collagen fibers

D In patients with abnormal or excessive scar tissue formation, treatment and prognosis will be driven by the correct diagnosis of a keloid versus a hypertrophic scar. This patient presents with a recurrent keloid of the chest. His clinical history supports this diagnosis by recurrence after resection, growth extending beyond the original border of the lesion, late recurrence after several years, and continued growth over several years without regression or improvement. Hypertrophic scars are less likely to recur, contained within the original boundaries of the lesion, often regress somewhat within a year, and recur earlier in the postoperative period if they are to recur. Both hypertrophic scars and keloid scars can be pruritic. Pathologic analysis of keloids reveals more type I collagen than type III collagen, similar to normal skin. Hypertrophic scars will exhibit increased type III collagen and pro-fibrotic collagen cross-linking. Keloid growth is thought to be impacted by cell-signaling between keratinocytes and fibroblasts, but hypertrophic scar production requires an abundance of myofibroblasts expressing smooth muscle actin. While hypertrophic scars have parallel collagen fibrils and bundles, keloids are characterized histologically by thick, randomly oriented collagen fibrils that are not organized into bundles.

A 65-year-old man undergoes microvascular reconstruction with a radial forearm flap following total laryngectomy and partial pharyngectomy for a T4 N3 M0 laryngopharyngeal squamous cell carcinoma. Postoperatively, the patient reports perioral numbness and tingling. Which of the following electrolyte abnormalities is most likely in this patient? A) Hypercalcemia B) Hyperkalemia C) Hypernatremia D) Hypocalcemia E) Hyponatremia

D Patients who undergo laryngopharyngectomy often have the thyroid gland removed as part of the operation. Sometimes, the parathyroid glands are removed accidentally or the blood supply is transiently interrupted, causing poor function. The parathyroid glands regulate calcium metabolism, and injury or removal of these glands manifests first as perioral tingling and numbness before more severe symptoms such as tetani. Hypernatremia is incorrect because, when symptomatic, it causes nausea, vomiting, and confusion. Hyponatremia is incorrect because, when symptomatic, it causes nausea, vomiting, lethargy, and confusion. Hypocalcemia is the correct answer because it is a concern for any patient who has this operation and reconstruction. Perioral tingling and numbness is an early sign of problems. Hypercalcemia is incorrect because, when symptomatic, it causes gastrointestinal issues such as vomiting, diarrhea, constipation, and skeletal muscle pain. Hyperkalemia is most likely to cause EKG changes and metabolism is not associated with parathyroid function.

A 58-year-old man undergoes total laryngopharyngectomy for recurrent squamous cell carcinoma. The pedicle to the most appropriate flap for reconstruction of the resulting total circumferential pharyngectomy defect extending from the base of the tongue to the cervical esophagus is located between which of the following muscles? A) Flexor carpi radialis and palmaris longus B) Teres minor, teres major, and long head of the triceps C) Teres minor, teres major, long head of the triceps, and humerus D) Vastus lateralis and rectus femoris E) Vastus medialis and rectus femoris

D The best option for reconstruction in this patient requiring circumferential pharyngeal reconstruction is the anterolateral thigh flap. This fasciocutaneous flap is supplied by perforators from the descending branch of the lateral femoral circumflex vessels, which are a branch of the profunda femoris vessels. The descending branch runs between the vastus lateralis and rectus femoris muscles, not the vastus medialis and rectus femoris. The radial forearm flap is based on the septum between the flexor carpi radialis and brachioradialis muscles in the arm. Although it can be used to reconstruct partial, noncircumferential pharyngectomy defects, it is not ideal for a long, circumferential defect in a previously radiated neck. The pedicle runs between the flexor carpi radialis and brachioradialis, not the palmaris longus. The circumflex scapular artery emerges from the triangular space in the back, which is defined by the teres minor, teres major, and the long head of the triceps. It is the pedicle to the parascapular and scapular flaps. Option C defines the quadrangular space that transmits the axillary nerve and posterior humeral circumflex artery.

A 50-year-old man with a long-term history of sun exposure is evaluated because of a 2.5-cm biopsy-proven squamous cell carcinoma of the central lower lip. Physical examination shows no enlarged cervical lymph nodes. Which of the following is the most appropriate treatment for this cancer? A) Combined surgery and radiation B) Curettage and electrodessication C) Photodynamic therapy D) Surgical excision E) Treatment with 5-fluorouracil for 3 months

D The cancer described in the scenario is a stage T2N0M0 cancer of the lip. Surgical excision with margin control is the most appropriate and probably the most common treatment for this lesion, although Mohs micrographic surgery may also be an option for surgical removal. Reconstruction would be performed after attaining adequate surgical margins and can be performed utilizing local tissue flaps in most patients. 5-Fluorouracil cream can be used in the treatment of actinic keratosis and superficial basal cell cancers. It is also used in the treatment of some squamous cell cancers, but its efficacy is lower and would not be indicated for a T2 cancer. In curettage and electrodessication, small tumors are scraped off with a curette and electrocautery destroys residual tumor and controls bleeding. It is used for superficial squamous cell cancers without high-risk characteristics in noncosmetically sensitive sites and would not be appropriate for this lesion. Also, surgical margins cannot be evaluated with this technique. Photodynamic therapy involves application of a photosensitizing agent, such as 5-aminolevulinic acid (5-ALA), that is taken up by the cancer cells. The next day, the medicated areas are activated by a strong light. The treatment selectively destroys cancer cells while causing minimal damage to the surrounding tissue. It is used to treat actinic keratosis and is not approved by the Food and Drug Administration for treatment of squamous cell carcinoma, although experience suggests that it is effective for small, superficial cancers. Radiation therapy alone can be used in cosmetically and functionally important sites such as the lip or in patients who are poor candidates for surgery because of medial comorbidities. Given the good functional and cosmetic prognosis of surgical resection and reconstruction in t

A 42-year-old man is evaluated because of a 2-cm mass in the anterior floor of the mouth. Examination shows no palpable masses in the neck. In addition to surgical tumor removal, which of the following is the most appropriate additional step in management? A) Radiation only B) Radical neck dissection C) Radical neck dissection and radiation D) Selective neck dissection E) Observation

D The correct response is Option D. A 2-cm mass with a clinically negative neck, pT2cN0, merits a supraomohyoid neck dissection because of the high risk of occult spread in this zone. A supraomohyoid dissection removes the lymph nodes in zones I, II, III; a modified neck dissection covers I-IV. A smaller tumor, pT1N0, can be managed with a "tight 'wait and watch'" strategy. A radical neck dissection includes the sternocleidomastoid muscle and is not necessary for a clinically negative neck. Adding radiation therapy to a reflex neurovascular dystrophy is also not necessary for an N0 neck.

Which of the following cranial nerves develops with the first branchial arch? A) Facial (VII) B) Glossopharyngeal (IX) C) Hypoglossal (XII) D) Trigeminal (V) E) Vagus (X)

D The first branchial arch is associated with the trigeminal nerve (V), second branchial arch with the facial nerve (VII), third branchial arch with the glossopharyngeal (IX), fourth branchial arch with the superior laryngeal (X), and sixth branchial arch with the recurrent laryngeal (X).

During recipient vessel dissection in preparation for a head and neck free flap reconstruction, a nerve that runs deep and roughly parallel to the posterior belly of the digastric muscle is accidentally divided. This patient is most likely to have which of the following dysfunctions as a result? A) Impaired shoulder elevation B) Loss of depressor anguli oris movement C) Paralysis of the hemidiaphragm D) Tongue deviation E) Vocal cord paralysis

D The hypoglossal (XII) nerve emerges from the hypoglossal canal in the skull base and runs deep and roughly parallel to the posterior belly of the digastric in the upper neck, eventually innervating the intrinsic and extrinsic muscles of the tongue (except for the palatoglossus, which is innervated by the vagus [X] nerve). Division of the nerve results in ipsilateral tongue paralysis. The posterior belly of the digastric muscle is known as the "resident's friend" because it acts as a useful anatomic landmark for several critical structures in the neck. It lies directly superficial to the branches of the external carotid artery, the internal carotid artery, jugular vein, and hypoglossal (XII) nerve. The marginal mandibular nerve, a branch of the facial (VII) nerve, innervates the depressor anguli oris and depressor labii inferioris, which are the facial mimetic muscles responsible for frowning. It runs superficial to the posterior belly of the digastric muscle but deep to the platysma muscle. It is often injured in surgery of the neck region. The vagus (X) nerve exits the cranium via the jugular foramen and courses from superior to inferior within the carotid sheath between the internal jugular vein and common carotid artery. It has diverse functions including supplying the larynx, pharynx, heart, esophagus, stomach, and small and large intestines. Transection of the vagus (X) nerve in the neck can result in vocal cord paralysis, among other dysfunctions. The spinal accessory (XI) nerve innervates the sternocleidomastoid and trapezius muscles. Therefore, its division results in impaired shoulder mobility. Although it lies deep to the posterior belly of the digastric muscle, it courses inferiorly and posteriorly, crossing the internal jugular vein, rather than parallel to the muscle, to enter the posterior triangle of the neck. The phrenic nerve originates from cervical spinal nerves (C3-C5) and is found in the floor of the posterior triangle of the neck and is not found deep to the posterior belly of the digastric.

A 51-year-old woman is scheduled to undergo a lower lip reduction. Blockade of the mental nerve is planned for anesthesia. The most appropriate site for injection of the mental foramen blockade is the mucosa below which of the following teeth? A) Canine B) Central incisor C) Lateral incisor D) Second bicuspid E) Second molar

D The mental nerve foramen is located near the second bicuspid or first molar along the border of the mandible. This nerve will give sensation to the lower lip. The other answers are too distal or mesial for the mental nerve foramen.

A 30-year-old woman with cerebral palsy has excessive salivation at rest (unstimulated) that is not controlled with administration of glycopyrrolate (Robinul). Removal of which of the following salivary glands is most likely to reduce salivary flow in this patient? A ) Minor B ) Parotid C ) Sublingual D ) Submandibular

D The parotid and submandibular glands are the main contributors to salivary flow. Minor salivary glands are present in the oral cavity and pharynx. They are minor contributors, creating less than 10% of the saliva. The secretory unit of the salivary glands is constructed of acinar cells, myoepithelial cells, intercalated duct, striated duct, and excretory duct. The acini are responsible for secreting serous and mucous constituents of saliva. The parotid gland is purely a serous-secreting gland, whereas the submandibular is predominately serous, with 10% of the acinar cells producing mucous secretions. In the unstimulated state, the submandibular gland produces most of the saliva, whereas the parotid gland is responsible for most of the saliva produced in the stimulated state. Total salivary flow can reach 1.5 L daily in healthy individuals. Xerostomia is a common complication resulting from radiation therapy for head and neck cancer. Xerostomia is defined as dry mouth (reduced or absent saliva flow) caused by damage to the salivary glands. Xerostomia has late effects on oral health, specifically dry mouth, sore throat, altered taste, dental decay, changes in voice quality, and impaired chewing and swallowing function. Xerostomia may also contribute to the development of mandibular osteoradionecrosis after radiation. Salivary flow reduces to 50 to 70% of baseline after 10 to 16 Gy radiation and is undetectable after 40 to 42 Gy radiation. Xerostomia has been reported to occur in 60 to 90% of survivors of head and neck cancers treated with radiation therapy. Management of xerostomia is focused on prevention and treatment. Although there are multiple options and advances that have been made in the management of this condition, there are no specific regimens that will prevent or completely treat xerostomia. Prevention o

A 39-year-old woman undergoes a total parotidectomy with facial nerve preservation for mucoepidermoid carcinoma of the parotid gland. The final pathology report indicates microscopic disease at the deep margin, and follow-up imaging shows no gross residual disease. No detectable nodal or other metastases are noted. Which of the following is the most appropriate next step in management? A) Chemotherapy B) Immunotherapy C) Neck dissection D) Radiation therapy E) Reexcision of the deep margin

D The patient described likely has a stage III tumor (T3 N0 M0). Standard management algorithms developed by the National Comprehensive Cancer Network recommend adjuvant radiation treatment when the persistence of positive margins relates to microscopic disease and not gross disease. If there is gross disease, either by physical examination or follow-up imaging, and it is resectable, then surgical resection of the residual disease should be done initially, followed by adjuvant radiation. Chemotherapy for major salivary gland tumors is appropriate as a first-line therapy concomitant with radiation only in cases of squamous cell carcinoma. In patients with mucoepidermoid, adenoid cystic, and adenocarcinomas, the role of chemotherapy is mainly palliative and reserved for advanced situations of recurrent or distant systemic disease. The absence of standard chemotherapy protocols for these situations attests to the degree of response that can be expected. Immunotherapy has no significant role in the treatment of major salivary gland malignancies. Neck dissection is indicated for malignant salivary gland tumors with clinically positive nodes detected either on physical examination or with preoperative imaging workup. This applies to parotid tumors of either the superficial or the deep lobe. Typical imaging to identify nodal disease would include CT scan or MRI or both. Neck dissections performed electively are rarely indicated, and only in very high-risk situations that are based on factors other than clinical and histologic features of the primary tumor. Radiation treatment is an effective treatment for negative necks with high risk of nodal disease, and is preferred over elective neck dissections. Surgical resection of persistent disease is indicated when a previously treated parotid mass was incompletely resected, and th

During embryological development, which of the following structures is derived from the same pharyngeal arch as the antihelix? A) Mandible B) Maxilla C) Mylohyoid D) Stapes E) Tensor veli palatini

D The pharyngeal arches are created during embryological development by the migration of the neural crest cells and the surrounding pharyngeal endoderm and mesoderm. The first four pharyngeal arches are the most prominent and have unique nervous, arterial, muscular, and bony components. The muscular components of the first pharyngeal arch are innervated by the trigeminal nerve (cranial nerve V) and include the muscles of mastication, as well as the anterior digastric, mylohyoid, tensor tympani, and tensor veli palatini. The maxillary artery supplies blood to the components of the first pharyngeal arch. The greater wing of the sphenoid, as well as the incus, the malleus, maxilla, zygomatic, mandible, and temporal bones comprise the bony components of the first pharyngeal arch. The second pharyngeal arch is innervated by the facial nerve (cranial nerve VII) and receives blood supply from the stapedial artery. The bony components of the second pharyngeal arch include the stapes, styloid process, stylohyoid ligament, lesser horn, and upper body of the hyoid. The muscular components of the second pharyngeal arch include the muscles of facial expression, as well as the posterior digastric, stylohyoid, and stapedius.During embryological development of the external ear, the antihelix, antitragus, and lobule are derived from the three posterior hillocks of the second pharyngeal arch, while the tragus, the root of the helix, and the superior helix are derived from the three anterior hillocks of the first pharyngeal arch.

Which of the following cranial nerves develops within the first branchial arch? A) Facial (VII) B) Glossopharyngeal (IX) C) Hypoglossal (XII) D) Trigeminal (V) E) Vagus (X)

D The trigeminal nerve (cranial nerve V) develops from the first branchial arch and gives rise to the malleus and incus. The ligaments associated with the first branchial arch are the anterior ligament of the malleus and the sphenomandibular ligament. The muscles of the first branchial arch include the muscles of mastication (masseter, temporalis, medial pterygoid, lateral pterygoid), tensor veli palatini, tensor tympani, mylohyoid, and the anterior belly of the digastric. The first pouch is associated with the external auditory canal and middle ear space.The facial nerve (cranial nerve VII) develops from the second branchial arch and gives rise to the stapes, styloid, and the upper body of the hyoid. It includes the stylohyoid ligament. The associated muscles are facial expression muscles, stapedius, stylohyoid, and the posterior belly of the digastric. The second pouch is associated with the tonsillar fossa.The glossopharyngeal nerve (cranial nerve IX) develops from the the third branchial arch and gives rise to the lower body of the hyoid. Its muscle is the stylopharyngeus, and the pouch is associated with the inferior parathyroid gland and the thymus.The fourth branchial arch is associated with the larynx. The muscles associated with this arch are the laryngeal, pharyngeal, and soft palate. The pouch is associated with the superior parathyroid gland and the thyroid gland.

An 18-year-old woman is referred for evaluation because of bulging of the right upper eyelid that has progressed slowly for the past 6 years. Physical examination shows soft enlargement of the right upper eyelid with ptosis; skin color shows no abnormalities and vision is preserved. CT scan shows no intraorbital, intracranial, or bony involvement. Examination of a specimen obtained on biopsy shows spindle cells and mast cells in a collagenous myxoid stroma. Which of the following is the most likely diagnosis? A ) Cobb syndrome B ) Fibrous dysplasia C ) Klippel-Trénaunay syndrome D ) Neurofibroma E ) Sturge-Weber syndrome

D These orbitofacial tumors may be part of a syndrome of neurofibromatosis 1 (NF1), which is an autosomal dominant disorder. When the full syndrome is present, skeletal dysplasia such as absence of the greater wing of the sphenoid (5 to 7%) and macrocephaly may be present. However, solitary neurofibromas unassociated with other features are not uncommon. There are three subtypes of neurofibromas: localized, plexiform, and diffuse. About 30% of plexiform neurofibromas outside of the central nervous system are associated with NF1, and the remainder are isolated. The tumors of NF2 are nearly all confined to the central nervous system and are similar to schwannomas. The scenario describes a case of plexiform neurofibroma. The majority of plexiform neurofibromas are present at birth, and half are found in the head and neck region. These tumors tend to grow in the prepubescent ages as a result of hormonal stimuli. Pathology is distinctive, and there is a 13% rate of malignant transformation. Surgical debulking and reconstruction remains the best treatment option. Cobb syndrome consists of a capillary malformation in the midline scalp region overlying an encephalocele, or in the skin posterior to an area of dysraphism in the cervical or lumbosacral spine. It does not involve the face or orbit. Fibrous dysplasia is an overgrowth syndrome of bones only, due to abnormal proliferation of bone-forming mesenchyme. It does not involve soft-tissue hypertrophy, nor is it associated with vascular malformations. Albright syndrome is a specific variety of polyostotic fibrous dysplasia and includes endocrine abnormalities and café-au-lait spots. Klippel-Trénaunay syndrome is a capillary-lymphatic-venous malformation typically involving hypertrophy of the extremities and sometimes the thorax of one side of the body, and does not involve

A 65-year-old woman is evaluated immediately after she underwent subtotal maxillectomy for an adenoid cystic carcinoma. The resulting defect includes the right hemi-palate, maxillary sinus, orbital floor, and nasal lining. A photograph is shown. Which of the following is the most appropriate option for reconstruction? A) Facial artery musculomucosal flap B) Obturator C) Omentum flap D) Rectus flap E) Temporalis flap

D This is an extensive subtotal maxillectomy (Type IIIA Cordeiro classification) that requires reconstruction of a number of anatomical structures including the hard palate, the orbital floor, and the nasal lining. In addition, vascularized tissues are required to separate the maxillary sinus content from the orbit. In this case, the rectus flap is the best option because it can provide skin to re-line the maxillary sinus and repair the palatal defect. In addition, the rectus flap has sufficient bulk to obliterate the maxillary sinus and provide vascularized tissue to support the orbital floor reconstruction that is required. The omentum flap usually has enough bulk to obliterate the sinus but will not enable reconstruction of the palatal defect. The obturator will reconstruct the palate but will not provide vascularized tissue to line the orbital floor reconstruction and separate this region from the maxillary sinus. The temporalis flap can be used to cover the orbital floor reconstruction but does not reconstruct the palatal defect. The facial artery musculomucosal flap is a flap based on the facial artery and would not be useful in this reconstruction.

A 32-year-old man comes to the office because he has had painless swelling of the right side of the jaw for the past 4 months. Examination shows a 6-cm mass over the mandibular angle. Panoramic x-ray study (Panorex) of the mouth is shown. Examination of the specimens obtained on biopsy of the lesion shows palisading odontogenic cells. Which of the following is the most appropriate management? A ) Curettage and cancellous bone graft B ) Curettage only C ) Induction chemotherapy followed by irradiation D ) Segmental resection and immediate reconstruction with a vascularized bone graft E ) Segmental resection and nonvascularized bone graft

D This patient has an ameloblastoma, which is a rare cystic tumor involving the mandible. Characteristic findings include a multilocular radiolucent lesion with a ?soap bubble? appearance, usually in association with an impacted molar. X-ray studies show unilocular or multilocular cystic masses associated with thinning of the surrounding bone. Examination of a biopsy specimen shows palisading odontogenic cells. Appropriate treatment consists of segmental mandibular resection, including a margin of normal bone and any adjacent teeth, and immediate reconstruction. Immediate reconstruction will rapidly restore facial function and improve facial aesthetics. Curettage is inadequate and is associated with a recurrence rate of 50 to 100%. Ameloblastomas are generally benign. Malignant forms of ameloblastoma may occasionally be treated with adjuvant radiation therapy in addition to surgical resection.

A 37-year-old man is waiting for a facial deceased-donor vascularized composite allotransplant. Which of the following components of a total face transplant is most antigenic? A) Bone marrow B) Muscle C) Nerve D) Skin E) Tendon

D Vascularized composite allotransplantation refers to the transplantation of an allograft consisting of heterogeneous cadaveric tissues. It provides a means of restoring structural, functional, and aesthetic form in severely injured patients. The potential for improvements in quality of life must be offset by the need for lifelong immunosuppression in adults with non-life-threatening injuries. The benefits and difficulties of immunosuppressive drugs have been established in solid organ transplantation. Regimens derived from renal transplantation have been successfully applied to composite tissue allografts. Overall, more than 60 hand/forearm/arm transplantations and 16 face transplantations have been performed in the past 12 years. The overall functional and aesthetic outcome is satisfactory, but side effects and complications related to immunosuppression are challenges hindering progress in this field. The high levels of immunosuppression, skin rejection, nerve regeneration, donor legislation, and the acceptance level need to be addressed to promote growth of this promising new field in transplantation and reconstructive surgery. Because composite tissue allograft transplantations are not life-saving procedures, much consideration is devoted to the issue of minimizing or withdrawing immunosuppression. When compared with solid organ transplants, composite tissue allografts are histologically heterogeneous, composed of different tissue types (e.g., skin, muscle, bone, bone marrow, lymph nodes, nerve, and tendon) and express different immunogenicity of transplanted elements. The hierarchy of antigenicity of composite tissue allografts was introduced in an experimental model of limb transplantation and showed that skin is the most antigenic tissue, and together with muscle, subcutaneous tissue, and bone (including bone

A 72-year-old man presents with a 2-cm, nontender, rapidly growing, violaceous intradermal papule of the cheek. Current medications include tacrolimus following renal transplantation. Biopsy of the lesion shows small, round, blue cells with large prominent nuclei that stain positive for cytokeratin 20. Which of the following is the most likely diagnosis? A) Amelanotic melanoma B) Basal cell carcinoma C) Keratoacanthoma D) Merkel cell carcinoma E) Squamous cell carcinoma

D Merkel cell carcinoma (MCC) is an uncommon and extremely aggressive cutaneous malignancy that is challenging to diagnose. Up to one half of patients will eventually develop a recurrence or a metastasis. There are approximately 2500 cases of MCC diagnosed per year in the United States. Eighty percent of MCCs are caused by the Merkel cell polyomavirus and the remaining 20% by extensive ultraviolet-mediated damage.MCC is most common on sun-damaged areas, with half located on the head and neck and nearly 40% on the extremities. They usually present as nontender, rapidly growing, painless, single, red to violaceous, firm intradermal papules or nodules. The epidermis overlying the tumor is usually preserved, but ulceration or crusting is not uncommon. Their doubling time can be as short as 12 days. The clinical features of MCC are summarized in the acronym AEIOU: asymptomatic, expanding, immunosuppressed, older than 50 years, and ultraviolet-exposed fair skin. They are differentiated histologically by the small, round, blue cells that stain positive for cytokeratin 20.The surgical treatment of MCC consists of wide local excision with one to two centimeter margins, inclusive of the underlying fascia. Management of regional disease is critical with this tumor. A clinically negative nodal basin will be evaluated with a sentinel node biopsy while a patient with a clinically positive nodal basin will be offered a therapeutic complete lymph node dissection. Postoperative radiation is offered to patients with tumors greater than 1 cm, close/positive margins, or nodal involvement.Melanoma cells stain positive for S100 and HMB-45. Basal cell carcinomas stain positive for Ber-EP4. Squamous cell carcinomas stain positive for AE1/AE3.

For total laryngopharygeal defects, which of the following is an advantage of performing reconstruction with the jejunal free flap instead of the anterolateral thigh free flap? A) Better voice production with tracheoesophageal puncture prosthesis B) Greater feeding-tube independence C) Less donor site morbidity D) Lower flap loss rates E) More straightforward inset

E Both the jejunal free flap and the anterolateral thigh (ALT) free flap have been used to restore continuity of the hypopharynx and cervical esophagus following laryngopharyngectomy and have shown superior functional results and lower complication rates that previously used pedicled flap techniques. The jejunal free flap has become less popular in many centers in recent years because the ALT free flap has lower donor site morbidity, a higher rate of feeding tube independence, and equivalent or lower flap loss rates. Additionally, voice production with a tracheoesophageal puncture (TEP) prosthesis is considered superior with the ALT. The TEP speech with the jejujunal flap is characterized as wet and cavernous and, therefore, more difficult to understand. As a pre-formed tube, the jejunal flap is more straightforward to inset. The surgeon only has to perform a superior (base of tongue) and inferior (esophagus) mucosal anastomosis, whereas the ALT is a rectangular skin flap that must be tubed. The additional (vertical) suture line theoretically raises the risk for fistula formation. In a comparative study, Yu et al found the fistula rate to be 8% for the ALT free flap and 3% for the jejunal free flap. While the jejunum has inherent peristaltic activity, some studies have suggested that this movement does not improve bolus transit, but instead contributes to regurgitation and dysmotility.

A 16-year-old boy comes to the office because of a progressive 6-month history of unilateral nasal obstruction and frequent epistaxis. Anterior rhinoscopy shows a soft, smooth, purplish lobulated mass filling the left nasal cavity. An attempted office biopsy results in profuse bleeding. Which of the following is the most likely diagnosis? A) Dermoid cyst B) Encephalocele C) Hemangioma D) Inverted papilloma E) Nasopharyngeal angiofibroma

E Nasopharyngeal angiofibromas, also known as juvenile nasopharyngeal angiofibromas, are benign but locally invasive vascular tumors that occur almost exclusively in adolescent males. Onset is most common in the second decade of life, and rarely occurs after age 25 years. Symptoms include unilateral or bilateral nasal obstruction, frequent epistaxis or blood-tinged nasal discharge, and conductive hearing loss from Eustachian-tube obstruction. In advanced stages, the angiofibroma can deform the nose, face, and orbits, as well as erode into the cranial cavity and put pressure on the optic chiasm, resulting in diplopia. Treatment is usually surgical with radiation and reserved for extensive cases such as those with intracranial extension. Preoperative embolization as well as hormone therapy with estrogens, may limit blood loss. Nasopharyngeal angiofibromas are highly vascular, and office biopsies should be avoided. Inverted papilloma is a benign, locally aggressive neoplasm that arises in the nasal cavity and is associated with squamous cell carcinoma in approximately 5% of patients. The age of onset is usually between 40 and 60 years. Surgery is the primary treatment of inverted papilloma. Encephaloceles are neural tube defects that result in sac-like protrusions of the meninges (meningocele) or brain and meninges (meningoencephalocele) in various locations along the cranium, including intranasally. They tend to be bluish, soft, compressible masses that transilluminate. Biopsy may result in a cerebrospinal fluid leak. Hemangiomas are benign vascular lesions that are present at birth and characterized by a rapid growth phase around the age of 1 to 6 months followed by gradual involution over 1 to 12 years. A hemangioma would not be expected to first occur in adolescence. Dermoid cysts are derived from ectodermal and meso

A 45-year-old woman is evaluated for a 2.5-cm, biopsy-proven squamous cell carcinoma of the left floor of the mouth. Imaging studies show cortical mandibular invasion, but no enlarged cervical lymph nodes or distant metastatic disease. A tracheostomy is performed. Immediate reconstruction is planned. Which of the following is the most appropriate surgical treatment for this cancer? A) Wide local excision alone B) Wide local excision and marginal mandibulectomy C) Wide local excision and neck dissection D) Wide local excision, marginal mandibulectomy, and neck dissection E) Wide local excision, segmental mandibulectomy, and neck dissection

E Oral cavity cancers are staged based on the following criteria set forth by the American Joint Committee on Cancer: Primary tumor staging for oral cavity cancers (T) TX Primary tumor cannot be assessed T0 No evidence of primary tumor Tis Carcinoma in situ T1 Tumor = 2 cm in greatest dimension T2 Tumor >2 cm but not more than 4 cm in greatest dimension T3 Tumor >4 cm in greatest dimension T4a Moderately advanced, local disease Lip - Tumor invades through cortical bone, inferior alveolar nerve, floor of mouth, or skin of face Oral cavity - Tumor invades adjacent structures (e.g., through cortical bone, into deep extrinsic muscle of the tongue, maxillary sinus, or skin of face) T4b Very advanced, local disease Tumor invades masticator space, pterygoid plates, or skull base and/or encases internal carotid artery The cancer described is a stage T4aN0M0 cancer of the floor of mouth. The T-stage is 4a based on cortical mandibular invasion demonstrated by radiographic imaging. Concern for mandibular invasion should be raised whenever a tumor abuts or is fixed to the mandible. Mandibulectomy is indicated. Cortical invasion of the mandible is an indication for segmental mandibulectomy, in which the full thickness of the involved mandible and grossly disease-free margin are removed by osteotomies. Marginal mandibulectomy involves removal of the alveolar ridge and varying amounts of the inner or lingual table of the mandible depending on the location of the tumor. Marginal mandibulectomy is performed when cancers abut the mandible or invade the periosteum, but do not grossly invade the cortex of the bone. Although clinically and radiographically the neck does not have nodal metastases (stage N0), surgical treatment is usually performed due to the risk for occult nodal metastases (20% or more in some studies). Such dissectio

Which of the following best approximates the 3-year survival rate of osseointegrated dental prostheses used in reconstruction with a nonirradiated free fibula flap? A ) 5% B ) 25% C ) 50% D ) 75% E ) 95%

E Osseointegrated prostheses are currently used to anchor dental crowns, auricular and facial prosthetics, and hearing aids (bone-anchored hearing aids, or BAHAs). This technology has improved since it was first introduced, and long-term survival rates are generally good. Several studies have confirmed short to intermediate survival rates of around 95% for dental prostheses in nonirradiated free fibula flap reconstruction. The results are significantly worse in irradiated tissues.

A 46-year-old woman with no history of trauma has had 1 month of headaches and soft tissue swelling over the frontal sinus and lateral frontal bone. Today she has fever, mental status changes, and increased swelling and redness of the forehead. CT scan shows a frontal epidural abscess and opacified frontal sinuses. Which of the following is the most likely diagnosis? A) Exostosis B) Fibrous dysplasia C) Frontal osteoma D) Infected encephalocele E) Pott puffy tumor

E Pott puffy tumor is currently rare due to availability of antibiotics. Pott puffy tumor is an osteomyelitis and abscess that results from chronic frontal sinusitis. The clinical presentation and CT scan findings are typical for the presentation of this tumor. Fibrous dysplasia is a bony overgrowth tumor that is not infectious in nature. Encephalocele is a congenital condition with a skull defect. Exostosis and frontal osteoma are benign overgrowths of the bone that occur slowly. They are not associated with mental status changes.

A patient undergoes extraction of a fully impacted mandibular third molar. During corticotomy of the mandible, protection of the adjacent soft tissue is necessary to avoid injury to which of the following nerves? A) Facial B) Hypoglossal C) Inferior alveolar D) Infraorbital E) Lingual

E Protection of the lingual border of the mandible during extraction of mandible wisdom teeth is critically necessary because of the close proximity of the lingual nerve to the lingual border of the mandible. This nerve can be inadvertently injured if not routinely protected during third molar extractions.

A 65-year-old man with Stage II squamous cell cancer at the base of the tongue comes to the office. He has a 30-year history of smoking. Which of the following type of neck dissection is most appropriate for this patient? A) Modified: levels I to V nodes B) Radical: levels I to V nodes, CN XI, internal jugular vein, sternocleidomastoid muscle removed C) Selective (anterior compartment): VI D) Selective (lateral neck): levels II to IV E) Selective (supraomohyoid): levels I to III

E Selective removal of nodes in levels I to III is the procedure of choice for N0 and N1 cancers in the oral cavity. Lateral and anterior compartment dissections miss Level I, which is the closest lymphatic level to a base of tongue cancer. Modified neck dissections are indicated for clinically palpable metastatic disease. Radical neck dissections are indicated for involvement of the nerve, vein, or muscle.

An 80-year-old man presents with floor of mouth cancer on the right side. In addition to the 1-cm area of ulceration, there is a hard, warm, nonmobile, painful mass in the submandibular area. This mass most likely originated from which of the following types of tissue? A) Bone B) Fat C) Muscle D) Nerve E) Salivary

E The submandibular gland duct, also known as the Wharton duct, ends in the area of the floor of the mouth and is typically blocked when cancer invades this area. As a result, there is backup of salivary content, the gland enlarges, becomes firm and painful, and can even become infected. This event has no prognostic implications. However, an enlarged node, such as the perifacial nodes, can mimic this finding and can upstage a cancer patient. If a bimanual examination of the gland cannot be performed because of the patient's pain, a CT scan is easily confirmatory. In general, lymph nodes are mobile, with the rare exception of external bony erosion. Nonetheless, a complete head and neck physical examination can be key to interpreting the findings described. Bony expansion from a floor-of-mouth cancer is possible, but will not usually present as a painful mass. In general, fat is not involved. Nerve involvement can also occur, and perineural invasion in the lingual nerve can cause its enlargement, but not with this presentation. A sublingual infection or hematoma (e.g., Ludwig angina) can present with what is typically a swollen bilateral submandibular mass, as the infection/abscess extends below the mylohyoid muscle. This process can cause immediate airway compromise and many recommend an emergent tracheostomy. This pathology is almost always related to bacterial infections of the mouth, most commonly related to dental caries.

A 53-year-old man is undergoing revision of mandibular reconstruction. An iliac crest osteocutaneous free flap is planned. Which of the following is the main advantage of this flap compared with a free fibula osteocutaneous flap? A) Bone length B) Minimal donor site morbidity C) Pedicle length D) Reliability with multiple osteotomies E) Vertical height

E There are many choices for bone grafts, both vascularized and nonvascularized. In this clinical scenario, there is no question that a vascularized bone graft is indicated, given the irradiated field, anterior location, and >6-cm defect. The choice of vascularized bone grafts include the free fibula, free iliac crest, free scapula, and free radius bone grafts. All have their pros and cons individually, although collectively, vascularized bone free flaps provide 40% more strength, 56% more stiffness, higher complete arthrodesis rate, and superior functional outcomes. The free fibula flap would be the most common choice for this situation given its long pedicle (6 to 10 cm), the large amount of usable bone (22 to 24 cm), minimal donor site morbidity, ability to accept dental implants, and reliable skin flap (skin island survival rates approaching 100% due to increased anatomical understanding and improvements in harvest techniques). However, its disadvantage is that it doesn't have the vertical height of the free iliac crest.

A 55-year-old woman is evaluated because of a 2-year history of an enlarging right-sided facial mass. Examination shows a roughly 4-cm firm mass in the right parotid region and a firm lymph node in zone III of the ipsilateral neck. Imaging does not show metastatic disease. Fine-needle aspiration of the mass suggests high-grade mucoepidermoid carcinoma of the parotid gland. Which of the following is the most appropriate treatment in this patient? A) Neoadjuvant chemoradiation and reassessment of tumor response before additional treatment B) Superficial parotidectomy with ipsilateral cervical lymph node dissection C) Superficial parotidectomy with ipsilateral cervical lymph node dissection and postoperative adjuvant radiotherapy D) Total parotidectomy with ipsilateral cervical lymph node dissection E) Total parotidectomy with ipsilateral cervical lymph node dissection and postoperative adjuvant radiotherapy

E This patient has T2N1M0 (Stage II) high-grade mucoepidermoid carcinoma, and total parotidectomy with ipsilateral cervical dissection is the appropriate treatment. Because the patient is lymph node positive, both an ipsilateral cervical lymph node dissection and postoperative adjuvant radiotherapy are indicated. A superficial parotidectomy would perhaps be indicated in a low-grade mucoepidermoid carcinoma, but in a high grade lesion, total parotidectomy is more appropriate. Ipsilateral cervical lymph node dissection and postoperative adjuvant radiotherapy would be indicated in this patient, however. Although total parotidectomy and ipsilateral cervical lymph node dissection are appropriate, two factors make postoperative adjuvant radiotherapy an important component of treatment: the node positive status of the neck, and the high grade of the tumor. Chemotherapy remains somewhat controversial in the treatment of mucoepidermoid carcinoma, and, to date, is not a part of standardized therapy.

A 79-year-old woman undergoes excision and elective neck dissection of a 2.5-cm invasive squamous cell carcinoma of the right lateral surface tongue. She has no history of head and neck cancer, and there is no radiologic or clinical evidence of nodal or metastatic disease. Wide excision with adequate margins and ipsilateral modified radical neck dissection is performed. Elective neck dissection is most likely to result in which of the following outcomes in this patient? A) Decreased local recurrence B) Fewer postoperative complications C) Increased incidence of distant metastasis D) Increased nodal relapse E) Increased overall survival

E This patient will have increased overall survival compared with a patient who does not have elective neck dissection. The patient described has Stage II (T2 N0 M0) oral cancer (larger than 2 cm but not larger than 4 cm, has not spread to lymph nodes with no metastatic disease). There has been much debate regarding management of the neck in patients with early-stage oral cancers. The two primary options include elective neck dissection (ie, at the time of the primary tumor resection) versus therapeutic neck dissection in the case of nodal relapse. In a prospective, randomized, controlled trial study of patients with T1 or T2 node-negative oral squamous cell carcinoma, patients received either elective neck dissection at the time of primary tumor resection or therapeutic neck dissection after nodal relapse. At 3 years, patients who underwent elective neck dissection had a higher rate of survival compared with the therapeutic surgery group (69.5 vs. 45.9%, P<0.001). Patients who undergo elective neck dissection at the time of primary tumor resection have an increased number of postoperative complications and decreased nodal recurrence. Distant metastasis was the same between the two groups.

A 64-year-old man with a T3N2 oropharyngeal tumor of the tongue and tonsillar fossa undergoes transoral robotic surgery (TORS). Which of the following is the most likely advantage of TORS over traditional treatments? A) Patients undergoing TORS have less dysphagia B) Patients undergoing TORS have a lower risk of hemorrhage than with open approaches C) There is decreased incidence of postoperative infection D) There is no need for reconstruction with a free flap E) There is significant improvement in disease-free survival

E Transoral robotic surgery (TORS) has been demonstrated to be comparable or advantageous when compared with nonsurgical treatment. Overall and disease-free survival and quality of life are comparable between TORS patients and patients who received nonsurgical treatment. TORS patients had significantly higher disease-free survival than patients who underwent open approaches in a meta-analysis. Risk of hemorrhage after TORS is 6%, whereas the risk is less than 2% in open approaches. Risk of dysphagia and infection are not significantly different from traditional treatment. Functional outcomes are significantly less in TORS patients than in patients who received nonsurgical treatment. In one study, only 9% of TORS patients needed gastrostomy tube feeding 1 year post-procedure compared with 31% in the chemotherapy group. Depending on the size of the defects, patients will need either locoregional or free flaps (radial forearm and anterolateral thigh flaps are the most common) after TORS. The incidence of needing free flaps is lower in TORS, but free flaps are not obsolete in TORS. Contraindications to TORS include: tumor abuts carotid artery, resection will include greater than 50% of the tongue base or posterior pharyngeal wall, tumor is fixed to the prevertebral fascia, or midline tongue base tumors that can potentially cause injuries to both lingual arteries.

A 4-year-old boy is seen following a pitbull bite to his face. The periorbital findings are shown in the photograph. A stent is placed and the ends of the stent are advanced across the canaliculus. The ends of the stent will enter the nasal cavity in which of the following locations? A) Above the superior turbinate B) Below the superior turbinate C) Above the middle turbinate D) Below the middle turbinate E) Above the inferior turbinate F) Below the inferior turbinate

F Reconstruction of the lacrimal apparatus is a critical step in addressing this patient's periorbital wounds. Failure to properly manage this aspect of the injury will lead to epiphora and the inevitable need for a secondary procedure to manage tear drainage. Delayed reconstruction of this injury would likely require a conjunctivodacryocystorhinostomy, a surgically created conduit between the eyelid and the nose. The best initial management of this lacrimal apparatus disruption is as described—placement of a stent through the canaliculi, into the lacrimal duct (contained within the maxillary bone), and into the nose. The exit point of the nasolacrimal duct is via the valve of Hasner, below the inferior turbinate. It is here that the ends of the stent can be identified and retrieved, although this is often quite challenging.The frontal, maxillary, and anterior ethmoid sinus cells drain into the middle meatus, just below the middle turbinate. The sphenoid sinus and posterior ethmoid sinuses drain into the sphenoethmoid recess, between the nasal septum and the superior turbinate.


Kaugnay na mga set ng pag-aaral

Chapter 10: The South and Slavery

View Set

Test Review (2020) - Chapter 5 - The Skeletal System

View Set

AWS Solutions Architect Associate

View Set

How to Read Literature Like a Professor Vocab

View Set

Georgia Real Estate - Section 14 Unit 3

View Set